a series 1 - IAS Parliament

136
A SERIES 1 PRESTORMING TM 2020 MOCK TEST V - EXPLANATION INDEX QUESTION NO PAGE NO 1. .................................................................................................................................................. 5 2. .................................................................................................................................................. 6 3. .................................................................................................................................................. 6 4. .................................................................................................................................................. 8 5. .................................................................................................................................................. 8 6. .................................................................................................................................................. 9 7. .................................................................................................................................................. 9 8. ................................................................................................................................................ 10 9. ................................................................................................................................................ 11 10. ............................................................................................................................................ 12 11. ............................................................................................................................................ 14 12. ............................................................................................................................................ 15 13. ............................................................................................................................................ 17 14. ............................................................................................................................................ 19 15. ............................................................................................................................................ 21 16. ............................................................................................................................................ 22 17. ............................................................................................................................................ 25 18. ............................................................................................................................................ 26 19. ............................................................................................................................................ 27 20. ............................................................................................................................................ 28 21. ............................................................................................................................................ 30 22. ............................................................................................................................................ 31 23. ............................................................................................................................................ 33 24. ............................................................................................................................................ 34 25. ............................................................................................................................................ 35 26. ............................................................................................................................................ 36 27. ............................................................................................................................................ 38 28. ............................................................................................................................................ 39 29. ............................................................................................................................................ 41

Transcript of a series 1 - IAS Parliament

A SERIES

1

PRESTORMINGTM 2020

MOCK TEST V - EXPLANATION

INDEX

QUESTION NO PAGE NO

1. .................................................................................................................................................. 5

2. .................................................................................................................................................. 6

3. .................................................................................................................................................. 6

4. .................................................................................................................................................. 8

5. .................................................................................................................................................. 8

6. .................................................................................................................................................. 9

7. .................................................................................................................................................. 9

8. ................................................................................................................................................ 10

9. ................................................................................................................................................ 11

10. ............................................................................................................................................ 12

11. ............................................................................................................................................ 14

12. ............................................................................................................................................ 15

13. ............................................................................................................................................ 17

14. ............................................................................................................................................ 19

15. ............................................................................................................................................ 21

16. ............................................................................................................................................ 22

17. ............................................................................................................................................ 25

18. ............................................................................................................................................ 26

19. ............................................................................................................................................ 27

20. ............................................................................................................................................ 28

21. ............................................................................................................................................ 30

22. ............................................................................................................................................ 31

23. ............................................................................................................................................ 33

24. ............................................................................................................................................ 34

25. ............................................................................................................................................ 35

26. ............................................................................................................................................ 36

27. ............................................................................................................................................ 38

28. ............................................................................................................................................ 39

29. ............................................................................................................................................ 41

A SERIES

2

30. ............................................................................................................................................ 42

31. ............................................................................................................................................ 43

32. ............................................................................................................................................ 44

33. ............................................................................................................................................ 45

34. ............................................................................................................................................ 48

35. ............................................................................................................................................ 49

36. ............................................................................................................................................ 50

37. ............................................................................................................................................ 51

38. ............................................................................................................................................ 53

39. ............................................................................................................................................ 55

40. ............................................................................................................................................ 56

41. ............................................................................................................................................ 58

42. ............................................................................................................................................ 61

43. ............................................................................................................................................ 62

44. ............................................................................................................................................ 63

45. ............................................................................................................................................ 65

46. ............................................................................................................................................ 66

47. ............................................................................................................................................ 67

48. ............................................................................................................................................ 68

49. ............................................................................................................................................ 69

50. ............................................................................................................................................ 70

51. ............................................................................................................................................ 71

52. ............................................................................................................................................ 71

53. ............................................................................................................................................ 72

54. ............................................................................................................................................ 73

55. ............................................................................................................................................ 75

56. ............................................................................................................................................ 77

57. ............................................................................................................................................ 78

58. ............................................................................................................................................ 79

59. ............................................................................................................................................ 80

60. ............................................................................................................................................ 81

61. ............................................................................................................................................ 82

62. ............................................................................................................................................ 83

A SERIES

3

63. ............................................................................................................................................ 85

64. ............................................................................................................................................ 87

65. ............................................................................................................................................ 87

66. ............................................................................................................................................ 88

67. ............................................................................................................................................ 90

68. ............................................................................................................................................ 91

69. ............................................................................................................................................ 91

70. ............................................................................................................................................ 92

71. ............................................................................................................................................ 93

72. ............................................................................................................................................ 95

73. ............................................................................................................................................ 96

74. ............................................................................................................................................ 97

75. ............................................................................................................................................ 98

76. ............................................................................................................................................ 99

77. .......................................................................................................................................... 101

78. .......................................................................................................................................... 102

79. .......................................................................................................................................... 103

80. .......................................................................................................................................... 103

81. .......................................................................................................................................... 105

82. .......................................................................................................................................... 105

83. .......................................................................................................................................... 107

84. .......................................................................................................................................... 110

85. .......................................................................................................................................... 112

86. .......................................................................................................................................... 114

87. .......................................................................................................................................... 115

88. .......................................................................................................................................... 116

89. .......................................................................................................................................... 117

90. .......................................................................................................................................... 118

91. .......................................................................................................................................... 119

92. .......................................................................................................................................... 119

93. .......................................................................................................................................... 121

94. .......................................................................................................................................... 124

95. .......................................................................................................................................... 124

A SERIES

4

96. .......................................................................................................................................... 126

97. .......................................................................................................................................... 132

98. .......................................................................................................................................... 133

99. .......................................................................................................................................... 135

100. .......................................................................................................................................... 135

A SERIES

5

PRESTORMINGTM 2020

MOCK TEST V - EXPLANATION

1. Which one of the following technology works behind the establishment of Video

based Customer Identification Process (V-CIP), which will be a consent-based for

making the banks and other regulated entities to adhere the RBI’s know your

customer norms by leveraging the digital technology?

(a) Geo-Targeting

(b) Geo-Beaconing

(c) Geo-Tagging

(d) Geo-Framing

EXPLANATION:

The RBI amended the KYC norms allowing banks and other lending institutions

regulated by it to use Video based Customer Identification Process (V-CIP), a move which

will help them onboard customers remotely.

The V-CIP, which will be consent-based, will make it easier for banks and other regulated

entities to adhere to the RBI's Know Your Customer (KYC) norms by leveraging the digital

technology.

Live location of the customer (Geotagging) shall be captured to ensure that customer is

physically present in India.

Geotagging, or GeoTagging, is the process of adding geographical identification metadata

to various media such as a geotagged photograph or video, websites, SMS messages, QR

Codes or RSS feeds and is a form of geospatial metadata.So, option (c) is correct.

ADDITIONAL INFORMATION:

Geotargeting in geomarketing and internet marketing is the method of determining

the geolocation of a website visitor and delivering different content to that visitor

based on their location. This includes country, region/state, city, metro code/zip

code, organization, IP address, ISP or other criteria

Geo-Beaconing

Beacons are little physical objects (under two square inches, in most cases) that can

be placed in desired locations. Their sole purpose is to detect you, or more

specifically, your device, as you move into their range. The beacons themselves don’t

A SERIES

6

send content. Like geofencing, a signal is triggered when you’re near one, and a

server sends a push, text, in-app message, or even an email (though currently,

that’s a less likely application for beacons).

Beacons are sometimes referred to as BLEs, for Bluetooth Low Energy, which is

what fans of this technology like about it. The low energy bit means the batteries in

a beacon will last for a long time. The blueooth bit means that beacons can work

well for marketing even in areas with no WiFi or spotty cell service.

Bluetooth tech is low cost, and accessible to anyone with a smartphone, but it

requires the user to have bluetooth turned on.

Beacons aren’t designed to know you’re you, where you were a moment ago, or

where you’re going next. By and large, they’re just little stationary computers that

react: when your device moves near the beacon itself, it triggers a ping, and your

app—if you have an app with notifications enabled for the beacon’s particular

location—goes to work communicating with you. When you move out of that

beacon’s range, it’s basically a done deal.

Geo-Retargeting or Geo-Framing: This tactic is just done on mobile devices and is

a method of capturing mobile device IDs of users who have entered a defined

geographic location (Geo-Fence), AND targeting them with ads after they have left

the Geo-Fenced area, even if they never saw your ad while inside the Geo-Fence.

SOURCE:

https://www.livemint.com/industry/banking/rbi-introduces-video-based-

identification-process-for-kyc-11578598810566.html

2. Which of the following physical features spreads over maximum number of

countries?

(a) Andes Mountain

(b) Himalayan mountain ranges

(c) Nile river basin

(d) Amazon forest

3. Consider the following statements about the Microbial fuel cells:

1. It is a device that converts chemical energy to electrical energy by the action

of microorganisms.

2. It uses protozoa and viruses as the catalysts to oxidize organic and inorganic

matter and generates electricity.

A SERIES

7

3. These fuel cells can be used as batteries in Electric Vehicles.

Which of the statements given above are correct?

(a) 1 and 3 only

(b) 1 and 2 only

(c) 2 and 3 only

(d) 1, 2 and 3

EXPLANATION:

A microbial fuel cell (MFC) is a device that converts chemical energy to electrical energy

by the action of microorganisms.These electrochemical cells are constructed using either

a bioanode and/or a biocathode.So, statement 1 is correct.

A microbial fuel cell (MFC) is a bio-electrochemical system that drives an electric current

by using bacteria and a high-energy oxidant such as O2, mimicking bacterial interactions

found in nature.They are devices that use bacteria as the catalysts to oxidize organic and

inorganic matter and generate current. So, statement 2 is not correct.

These fuel cells can be used as batteries in Electric Vehicles.So, statement 3 is correct.

ADDITIONAL INFORMATION:

Microbial fuel cells

They are devices that use bacteria as the catalysts to oxidize organic and inorganic

matter and generate current.

Electrons produced by the bacteria are transferred to the negative terminal and flow

to the positive terminal.

Plants naturally deposit biomaterial as they grow which in turn feeds the natural

bacteria present in the soil.

This creates energy that can be harnessed by fuel cells and used to power a wide

range of vital conservation tools remotely, including sensors, monitoring platforms

and camera traps.

SOURCE:

https://indianexpress.com/article/explained/microbial-fuel-cells-technology-that-

enabled-a-fern-to-take-its-own-selfies-6071048/

A SERIES

8

4. In 1931, Mahatma Gandhi expressed his ideas of how the Indian constitution

should do in

(a) Common Wheel

(b) Young India

(c) Indian opinion

(d) Navjivan

5. With reference to the Indian Economy, consider the following statements:

1. When RBI purchases US Dollars from the Indian market, there is a possibility

of depreciation of Indian rupee.

2. When interest rates remain low to negative in the developed countries, there

may be fall in capital inflows to India.

Which of the statements given above is/are correct?

(a) 1 only

(b) 2 only

(c) Both 1 and 2

(d) Neither 1 nor 2

EXPLANATION:

The aggressive dollar buying by RBI has clearly checked the rupee’s strengthening amid

foreign fund inflows. Hence, When RBI purchases dollars, there is every possibility of

depreciation of rupee. So, statement 1 is correct.

When there are differences in real interest rates between two countries that allow for the

flow of financial capital, that capital flows to the country with the relatively higher real

interest rate and out of the country with the relatively lower real interest rate.When

interest rates remain low to negative in the developed countries, there may be an increase

in capital inflows to India. So, statement 2 is not correct.

SOURCE:

https://economictimes.indiatimes.com/markets/forex/how-rbis-dollar-buying-

impacts-rupee/articleshow/73025295.cms?from=mdr

A SERIES

9

6. In the context of the cultural history of India, a mudra called “Katakamukha

Hasta” has been a favourite of most of the classical dances. Which one of the

following statements best describes it?

(a) The fingers are held in such a manner as if to ask ‘why’

(b) Three fingers are joined to symbolise ‘Om’

(c) The fingers are held straight and close together like the ‘stop sign’

(d) All the finger are held straight and together except the thumb finger

EXPLANATION:

Katakaamukha means “Opening in a Bracelet”. Its done by bringing together the index

finger, middle finger and thumb. The ring finger and the little finger are raised at

angles.Katakaamukha is generally used to express the following

Plucking or picking flowers

Holding a necklace or a garland

Pulling the bow string

Talking and Seeing

To show preparing a paste of sandal or musk

Offering Beetle leaves

Here, it best describes Threefingers are joined to symbolise ‘Om’. So, option (b) is

correct.

7. With reference to the Constitution of India. consider the following statements:

1. No Parliamentary sanction is required for acquisition of any geographical

territory which is not a part of India.

2. The newly acquired territory from foreign nations shall remain as the ‘Union

Territory’ until the Indian Constitution is amended.

Which of the statements given above is/are correct?

(a) 1 only

(b) 2 only

(c) Both 1 and 2

(d) Neither 1 nor 2

EXPLANATION:

No Parliamentary sanction is required for acquisition of territory which is not part of

India. But admitting the acquired territory into union of india needs parlimentry

approval. So statement 1 is correct.

The newly acquired territory from foreign nations can remain as the Union Territory or

A SERIES

10

statebut only after constitution amendment. So statement 2 is not correct.

ADDITIONAL INFORMATION:

ARTICLE 2 : ADMISSION OR ESTABLISHMENT OF NEW STATES

Parliament may by law admit into the Union, or establish, new States on such terms and

conditions as it thinks fit.

ARTICLE 3 : FORMATION OF NEW STATES AND ALTERATION OF AREAS, BOUNDARIES

OR NAMES OF EXISTING STATES

Parliament may by law –

(a) form a new State by separation of territory from any State or by uniting two or more

States or parts of States or by uniting any territory to a part of any State;

(b) increase the area of any State;

(c) diminish the area of any State;

(d) alter the boundaries of any State;

(e) alter the name of any State:

Provided that no Bill for the purpose shall be introduced in either House of Parliament

except on the recommendation of the President and unless, where the proposal contained in

the Bill affects the area, boundaries or name of any of the States, the Bill has been referred

by the President to the Legislature of that State for expressing its views thereon within such

period as may be specified in the reference or within such further period as the President

may allow and the period so specified or allowed has expired.

SOURCE:

Indian Polity - Laxmikanth

8. Which one of the following epochs is the most recent?

(a) Paleocene

(b) Pliocene

(c) Oligocene

(d) Miocene

A SERIES

11

EXPLANATION:

So option (b) is correct.

9. Which one of the following statements best describes about the term “Synthetic

meat” which was seen in the news recently?

(a) It is a transgenic meat prepared in laboratory through ex-vivo method

(b) Totipotency stem cell culture method is preferred for its growth in labs

(c) It has an enhanced taste and nutrient value than the conventional meat

(d) It increases the carbon footprint by releasing more methane to the nature

EXPLANATION:

Cultured meat is meat produced by in vitro cell culture of animal cells, instead of from

slaughtered animals. It is a form of cellular agriculture. Besides cultured meat, the terms

slaughter-free meat, in vitro meat, vat-grown, lab-grown meat, cell-based meat, clean

meat, cultivated meat and synthetic meat have all been used by various outlets to

describe the product.So, option (a) is correct.

ADDITIONAL INFORMATION:

Cultured meat is produced using many of the same tissue engineering techniques

traditionally used in regenerative medicine.

A SERIES

12

The concept of cultured meat was popularized by Jason Matheny in the early 2000s

after co-authoring a seminal paper on cultured meat production and creating New

Harvest, the world's first non-profit organization dedicated to supporting in vitro

meat research.

In 2013, Mark Post, a professor at Maastricht University, was the first to showcase a

proof-of-concept for cultured meat by creating the first burger patty grown directly

from cells.

Since then, several cultured meat prototypes have gained media attention: however,

because of limited dedicated research activities, cultured meat has not yet been

commercialized, although several plants for the production of cultured meat are

already being built.

The production process still has much room for improvement, but it has advanced

under various companies.

Its applications lead it to have several prospective moral, health, environmental,

cultural, and economic considerations in comparison to conventional meat.

SOURCE:

https://recipes.timesofindia.com/articles/food-facts/7-facts-you-must-know-about-

synthetic-meat/photostory/66119864.cms

10. Consider the following statements:

1. Vedic farming is a natural agriculture free from pesticide and chemical

fertilizer.

2. Homoeo-farming involves the use of small potent substances to stimulate the

already existing potential of soil.

3. Natural eco-farming is done by recycling of farm waste and not by tilling the

land.

Which of the statements given above is/are correct?

(a) 1 and 3 only

(b) 3 only

(c) 2 and 3 only

(d) 1, 2 and 3

A SERIES

13

EXPLANATION:

Homeo is use of small potent substances to stimulate the already existing potential of

soil. So option 2 is correct.

Vedic farming is a natural agriculture free from pesticide and fertilizer. Vedic farming is a

farming done during vedic age.So option 1 is correct.

Natural farming is a system where the laws of nature are applied

to agricultural practices. This method works along with the natural biodiversity of each

farmed area, encouraging the complexity of living organisms, both plants, and animals

that shape each particular ecosystem to thrive along with food plants.

In natural farming, decomposition of organic matter by microbes and earthworms is

encouraged right on the soil surface itself, which gradually adds nutrition in the soil, over

the period.

In natural farming there no plowing, no tilting of soil and no fertilizers, and no weeding is

done just the way it would be in natural ecosystems.So option 3 is correct.

ADDITIONAL INFORMATION:

Similarities between natural farming and organic farming:

Natural and organic both are chemical free and more or less poison free farming

methods.

Both systems discourage farmers from using any chemical fertilizers, pesticides on

plants and in all agricultural practices.

Both farming methods encourage farmers to use local breeds of seeds, and native

varieties of vegetables, grains, pulses and other crops.

Organic and natural farming methods promote nonchemical and homemade pest

control methods.

Key differences between natural farming and organic farming:

In organic farming, organic fertilizers and manures like

compost, vermicompost, manure, etc. are used and added to farmlands from

external sources.

In natural farming, neither chemical nor organic fertilizers are added to the soil. In

fact, no external fertilizers are added to soil or give to plants whatsoever.

In natural farming, decomposition of organic matter by microbes and earthworms is

encouraged right on the soil surface itself, which gradually adds nutrition in the soil,

over the period.

A SERIES

14

Organic farming still requires basic agro practices like plowing, tilting, mixing of

manures, weeding, etc. to be performed.

In natural farming there no plowing, no tilting of soil and no fertilizers, and no

weeding is done just the way it would be in natural ecosystems.

Organic farming is still expensive due to the requirement of bulk manures, and it

has an ecological impact on surrounding environments; whereas, natural

agriculture is an extremely low-cost farming method, completely molding with local

biodiversity.

There are many working models of natural farming all over the world, the zero

budget natural farming (ZBNF) is the most popular model in India. This

comprehensive, natural, and spiritual farming system is developed by Padma

ShriSubhashPalekar.

SOURCE:

http://www.ugaoo.com/knowledge-center/what-is-natural-farming-how-is-it-different-

from-organic-farming/

11. Which of the following is/are the example(s) of the process of Symbiosis?

1. Lice on humans

2. Escherichia coli in gut of animals

3. Cuscuta on body of plants

Select the correct answer using the code given below.

(a) 2 only

(b) 1 and 3 only

(c) 2 and 3 only

(d) 1, 2 and 3

EXPLANATION:

Symbiosis is a close relationship between two species in which at least one species

benefits. For the other species, the relationship may be positive, negative, or neutral.

There are three basic types of symbiosis: mutualism, commensalism, and parasitism.

Lice on humans is an example of parasitism.

Escherichia coli in gut of animals is an example of mutualism.

Cuscuta on body of plants is an example of parasitism.

So, option (d) is correct.

A SERIES

15

ADDITIONAL INFORMATION:

Mutualism

Mutualism is a symbiotic relationship in which both species benefit. An example of

mutualism involves goby fish and shrimp. The nearly blind shrimp and the fish

spend most of their time together. The shrimp maintains a burrow in the sand in

which both the fish and shrimp live. When a predator comes near, the fish touches

the shrimp with its tail as a warning. Then, both fish and shrimp retreat to the

burrow until the predator is gone. From their relationship, the shrimp gets a

warning of approaching danger. The fish gets a safe retreat and a place to lay its

eggs.

Commensalism

Commensalism is a symbiotic relationship in which one species benefits while the

other species is not affected. One species typically uses the other for a purpose other

than food. For example, mites attach themselves to larger flying insects to get a “free

ride.” Hermit crabs use the shells of dead snails for homes.

Parasitism

Parasitism is a symbiotic relationship in which one species (the parasite) benefits

while the other species (the host) is harmed. Many species of animals are parasites,

at least during some stage of their life. Most species are also hosts to one or more

parasites.

Some parasites live on the surface of their host. Others live inside their host. They

may enter the host through a break in the skin or in food or water. For example,

roundworms are parasites of mammals, including humans, cats, and dogs. The

worms produce huge numbers of eggs, which are passed in the host’s feces to the

environment. Other individuals may be infected by swallowing the eggs in

contaminated food or water.

12. “Buddhacharita”, the biography of Buddha, was composed by which one of the

following poet?

(a) Ashvaghosa

(b) Soundaranatha

(c) Vatsyayana

(d) Bharavi

A SERIES

16

EXPLANATION:

Buddhacharita was composed by Ashvaghosa. So option (a) is correct.

ADDITIONAL INFORMATION:

There was political instability between the era of 500 years after the fall of the

Mauryan Empire and before the rise of the Gupta Empire. This period is marked as

the development of Dravidian languages and literature in the South India.

The Sanskrit language and various forms of Prakrit language developed and some

distinctive literature was written in these languages.

The two great Epics, the Ramayana and the Mahabharata, were compiled. Besides,

some of the Dharmasastra were also composed in this period.

Smritis

The Smritis have continued to play a very important role in Hindu life as it were

playing since the last two thousand years.

The Smritis explained the religious duties, usage, laws, and social customs.

The Smritis are the expanded version of the Dharmasutras, which covered the

period from (about) sixth century to third century B.C.

The Smritis compiled almost in the period of eight hundred years or even more.

Manusmriti is the oldest among all. It was composed in about the 1st century B.C.

Other important Smritis are −

o Naradasmriti,

o Vishnusmriti,

o Yajnavalkyasmriti,

o Brihaspatismriti, and

o Katyayanasmriti.

These Smritis (discussed above) are very important sources of law and social

customs of the contemporary society and hence, declared to be of divine origin.

Mahabhasya written by Patanjali was the most outstanding work in the field of

grammar during the period of 2nd century B.C.

The center of Sanskrit grammar learning shifted to the Deccan after Patanjali.

In Deccan, the Katantra School developed in the first century A.D. Sarvavarman

composed the grammar of Katantra. He was a great scholar in the court of Hala

(Satavahana King)

Katantra’s grammar was short and handy to help the learning of Sanskrit in about

six months.

A SERIES

17

Hala, the King of Satavahana wrote a great poetical work, namely ‘Gathasaptasati’ in

Prakrit language.

Asvaghosha was an important literary figure of this period. He was a great Buddhist

philosopher. He had written many poetry, plays, etc. Important of them are −

o Saundarananda,

o Buddhacharita,

o Vajrasuchi, etc.

‘Buddhacharita’ was written in the form of Mahakavya. It is a complete life of

Buddha. It has been translated into many languages of the world.

Remains of Asvaghosha's plays have been recovered from Turfan (in central Asia).

‘Svapnavasavadatta’ was written by Bhasa. It is famous Sanskrit play of the period.

The art of dance and drama had already been codified by Panini's time and

mentioned by Kautilya and Patanjali.

Natyashastra was written by Bharata in the same period.

‘Milindapanho’ was written in Pali language. It explains the Buddhist doctrines in

the form of a dialogue between Milinda and his teacher (the great Buddhist

philosopher Nagasena). Milinda is generally identified with the Indo-Greek king

Menander.

SOURCE:

Themes 2

13. Consider the following statements about the “World Food Programme”:

1. It is the world's largest humanitarian organization addressing hunger and

promoting food security.

2. It annually publishes the ‘Global Hunger Index’ to measure the extreme of

poverty and hunger.

3. It also works to improve and develop agriculture, forestry, fisheries, and land

and water resources.

Which of the statements given above is/are correct?

(a) 1 only

(b) 1 and 2 only

(c) 2 and 3 only

(d) 1 and 3 only

A SERIES

18

EXPLANATION:

The World Food Programme (WFP) is the food-assistance branch of the United Nations

and the world's largest humanitarian organization addressing hunger and promoting food

security. So, statement 1 is correct.

The Global Hunger Index report, prepared and published jointly by Irish aid agency

Concern Worldwide and German organization Welt Hunger Hilfewas, calculated on the

basis of four indicators that are - child mortality, undernourishment, child wasting

(weight for age) and child stunting. So, statement 2 is not correct.

The Food and Agriculture Organization of the United Nations (FAO) is a specialized

agency of the United Nations that leads international efforts to defeat hunger and

improve nutrition and food security. It helps governments and development agencies

coordinate their activities to improve and develop agriculture, forestry, fisheries, and land

and water resources. So, statement 3 is not correct.

ADDITIONAL INFORMATION:

World Food Programme

The World Food Programme (WFP) is the food-assistance branch of the United

Nations and the world's largest humanitarian organization addressing hunger and

promoting food security.

According to the WFP, it provides food assistance to an average of 91.4 million

people in 83 countries each year.

From its headquarters in Rome and from more than 80 country offices around the

world, the WFP works to help people who cannot produce or obtain enough food for

themselves and their families.

It is a member of the United Nations Development Group and part of its executive

committee

Goals and strategies

The WFP strives to eradicate hunger and malnutrition, with the ultimate goal in

mind of eliminating the need for food aid itself. Its objectives are to:

o "Save lives and protect livelihoods in emergencies"

o "Support food security and nutrition and (re)build livelihoods in fragile

settings and following emergencies"

A SERIES

19

o "Reduce risk and enable people, communities and countries to meet their

own food and nutrition needs"

o "Reduce under-nutrition and break the inter-generational cycle of hunger"

o "Zero Hunger in 2030"

WFP food aid is also directed to fight micronutrient deficiencies, reduce child

mortality, improve maternal health, and combat disease, including HIV and AIDS.

Food-for-work programmes help promote environmental and economic stability and

agricultural production

SOURCE:

https://www.wfp.org/overview

14. Which of the following are the important functions of the National Bank for

Agriculture and Rural Development (NABARD) of India?

1. Refinance to Rural Financial Institutions for investment credit.

2. Loans to State Governments for developing rural infrastructure.

3. Direct lending to Cooperatives and Producers’ Organization.

4. Direct lending to the individuals for developing warehousing infrastructure.

Select the correct answer using the code given below.

(a) 1 and 2 only

(b) 3 and 4 only

(c) 1, 2 and 3 only

(d) 1, 2, 3 and 4

EXPLANATION:

The following are the important credit related functions of the NABARD Bank (National

Bank for Agriculture and Rural Development)

Refinance to Rural Financial Institutions for investment credit (long term loan)

and production and marketing credit (short term loan) purposes for farm and off-

farm activities in rural areas.(statement 1 is correct)

Loans to State Governments for developing rural infrastructure and strengthening

of the Cooperative Credit Structure. (statement 2 is correct)

Loans for warehousing infrastructure to State Governments, State/ Central

government Owned/ assisted entities, Cooperatives, Federation of cooperatives,

Farmers’ Producers Organizations,(FPOs), Federations of Farmers’ Collectives,

Primary Agricultural Credit Societies (PACS) / Cooperative Marketing Societies

(CMS) or similar institutions, Corporates/ Companies, Individual entrepreneurs,

etc., (statement 4 is correct)

A SERIES

20

Direct lending to Cooperatives and Producers’ Organization, support to State

owned institutions /corporations under NABARD Infrastructure Development

Assistance and direct lending to individuals, partnership firms, corporates, NGOs,

MFIs, Farmers’ collectives etc. under Umbrella Programme for Natural Resource

Management (UPNRM)(statement 3 is correct)

Pass through agency of select Government of India Capital Investment Subsidy

Schemes.

So, option (c) is correct.

ADDITIONAL INFORMATION:

The major functions of NABARD include promotion and development, refinancing,

financing, planning, monitoring and supervision.

Non-credit related:

Credit Planning and Monitoring, Coordination with various agencies and

institutions.

Assist in policy formulation of GoI, RBI and State Governments on matters related to

agricultural credit and rural development

Institutional development and capacity building of Cooperatives and Regional Rural

Banks (RRBs) to strengthen the rural credit delivery system. Statutory inspection of

Regional Rural Banks (RRBs), State Cooperative Banks and District Central

Cooperative Banks (DCCBs), voluntary inspection of State Cooperative Agriculture

and Rural Development Banks (SCARDBs) and their off-site surveillance

Promotional and developmental initiatives in the areas of farm, off-farm, micro

finance, financial inclusion, Convergence with Govt sponsored programmes.

Supporting the financial inclusion efforts of Regional Rural Banks and Cooperative

Banks

Thrust on promotion of livelihood opportunities and Micro Enterprises

Capacity Building of Personnel and Board Members of Credit Cooperatives and Staff

of Rural Financial Institutions.

Support to research and development, rural innovations, etc.

SOURCE:

https://www.nabard.org/ftrcontent.aspx?id=492#:~:text=The%20major%20functions

%20of%20NABARD,with%20various%20agencies%20and%20institutions.

A SERIES

21

15. Which of the following is/are considered to be the primary scientific goals of

study at the world’s largest radio telescope “FAST”, which recently got it’s full-

fledged operation in China?

1. Pulsar observations

2. Detecting gravitational waves

3. Search for aliens

4. Neutral Hydrogen survey

Select the correct answer using the code given below.

(a) 2 only

(b) 4 only

(c) 1, 3 and 4 only

(d) 1, 2, 3 and 4

EXPLANATION:

The FAST website lists the following science objectives of the radio telescope:

Large scale neutral hydrogen survey

Pulsar observations

Leading the international very long baseline interferometry (VLBI) network

Detection of interstellar molecules

Detecting interstellar communication signals (Search for extraterrestrial

intelligence)

Pulsar timing arrays

So, option (c) is correct.

ADDITIONAL INFORMATION:

The Five-hundred-meter Aperture Spherical radio Telescope (FAST) is a radio

telescope located in the Dawodang depression, a natural basin in Pingtang County,

Guizhou, southwest China.

It consists of a fixed 500 m (1,600 ft) diameter dish constructed in a natural

depression in the landscape. It is the world's largest filled-aperture radio telescope,

and the second-largest single-dish aperture after the sparsely-filled RATAN-600 in

Russia.

A SERIES

22

It has a novel design, using an active surface made of metal panels that can be tilted

by a computer to help change the focus to different areas of the sky.

The cabin containing the feed antenna suspended on cables above the dish is also

moved using a digitally-controlled winch by the computer control system to steer the

instrument to receive from different directions. It observes at wavelengths of 10 cm

to 4.3 m.

SOURCE:

https://www.thehindu.com/news/international/World%E2%80%99s-largest-radio-

telescope-begins-operations/article14998964.ece

16. Recently, the Government of India has expanded the scope of “Pradhan Mantri

Garib Kalyan Package”. Which of the following are the key components of the

expanded scheme?

1. Free food Grains to the poor people.

2. Cash transfer to farmers under PM-KISAN Yojana.

3. Insurance scheme for health workers who are fighting COVID-19.

4. Free gas cylinders to the poor families.

Select the correct answer using the code given below.

(a) 1 and 2 only

(b) 1 and 4 only

(c) 1, 2, and 4 only

(d) 1, 2, 3 and 4

EXPLANATION:

The following are the key components of the expanded scheme

Insurance scheme for health workers fighting COVID-19 in Government Hospitals

and Health Care Centres

Cash transfer to farmers under PM KisanYojana - The first instalment of Rs 2,000

due in 2020-21 will be front-loaded and paid in April 2020 itself under the PM

KISAN Yojana.

Free Food Grains to the poor people - Government of India would not allow

anybody, especially any poor family, to suffer on account of non-availability of

foodgrains due to disruption in the next three months.80 crore individuals, i.e,

roughly two-thirds of India’s population would be covered under this scheme.

Free Gas cylinders to Poor families - Under PM Garib Kalyan Yojana, gas

cylinders, free of cost, would be provided to 8 crore poor families for the next three

A SERIES

23

months.

So, option (d) is correct.

ADDITIONAL INFORMATION:

PRADHAN MANTRI GARIB KALYAN PACKAGE

Insurance scheme for health workers fighting COVID-19 in Government Hospitals and

Health Care Centres

Safai karamcharis, ward-boys, nurses, ASHA workers, paramedics, technicians,

doctors and specialists and other health workers would be covered by a Special

insurance Scheme.

Any health professional, who while treating Covid-19 patients, meet with some

accident, then he/she would be compensated with an amount of Rs 50 lakh under

the scheme.

All government health centres, wellness centres and hospitals of Centre as well as

States would be covered under this scheme approximately 22 lakh health workers

would be provided insurance cover to fight this pandemic.

PM Garib Kalyan Ann (अन्न) Yojana

Government of India would not allow anybody, especially any poor family, to suffer

on account of non-availability of foodgrains due to disruption in the next three

months.

80 crore individuals, i.e, roughly two-thirds of India’s population would be covered

under this scheme.

Each one of them would be provided double of their current entitlement over next

three months.

This additionality would be free of cost.

Pulses:

o To ensure adequate availability of protein to all the above mentioned

individuals, 1 kg per family, would be provided pulses according to regional

preferences for next three months.

o These pulses would be provided free of cost by the Government of India.

Under Pradhan Mantri Garib Kalyan Yojana,

Benefit to farmers:

A SERIES

24

o The first instalment of Rs 2,000 due in 2020-21 will be front-loaded and paid

in April 2020 itself under the PM KISAN Yojana.

o It would cover 8.7 crore farmers

Cash transfers Under PM Garib Kalyan Yojana:

Help to Poor:

o A total of 20.40 crores PMJDY women account-holders would be given an ex-

gratia of Rs 500 per month for next three months.

Gas cylinders:

o Under PM Garib Kalyan Yojana, gas cylinders, free of cost, would be provided

to 8 crore poor families for the next three months.

Help to low wage earners in organised sectors:

o Wage-earners below Rs 15,000 per month in businesses having less than

100 workers are at risk of losing their employment.

o Under this package, government proposes to pay 24 percent of their monthly

wages into their PF accounts for next three months.

o This would prevent disruption in their employment.

Support for senior citizens (above 60 years), widows and Divyang:

o There are around 3 crore aged widows and people in Divyang category who

are vulnerable due to economic disruption caused by COVID-19.

o Government will give them Rs 1,000 to tide over difficulties during next three

months.

MNREGA

o Under PM Garib Kalyan Yojana, MNREGA wages would be increased by Rs

20 with effect from 1 April, 2020. Wage increase under MNREGA will provide

an additional Rs 2,000 benefit annually to a worker.

o This will benefit approximately 13.62 crore families.

Self-Help groups:

Women organised through 63 lakhs Self Help Groups (SHGs) support 6.85 crore

households.

Limit of collateral free lending would be increased from Rs 10 to Rs 20 lakhs.

Other components of PM Garib Kalyan package

Organised sector:

o Employees’ Provident Fund Regulations will be amended to include Pandemic

as the reason to allow non-refundable advance of 75 percent of the amount

or three months of the wages, whichever is lower, from their accounts.

A SERIES

25

o Families of four crore workers registered under EPF can take benefit of this

window.

Building and Other Construction Workers Welfare Fund:

o Welfare Fund for Building and Other Constructions Workers has been

created under a Central Government Act.

o There are around 3.5 Crore registered workers in the Fund.

o State Governments will be given directions to utilise this fund to provide

assistance and support to these workers to protect them against economic

disruptions.

District Mineral Fund

o The State Government will be asked to utilise the funds available under

District Mineral Fund (DMF) for supplementing and augmenting facilities of

medical testing, screening and other requirements in connection with

preventing the spread of CVID-19 pandemic as well as treating the patients

affected with this pandemic.

SOURCE:

https://pib.gov.in/PressReleaseIframePage.aspx?PRID=1608345

17. The term “Anthurium”, recently seen in the news, is

(a) a domestic flowering plant which can purify the surrounding air and remove

harmful airborne chemicals

(b) an enzyme in a plant which that plant releases to its damaged part so that a new

branch grows from that spot

(c) a lichen, also a indicator species, grown in the Eastern Himalayas

(d) a herb, grows in the Western Ghats, that is used by Kerala state government in the

treatment of COVID-19 patients

EXPLANATION:

It is a vast group of beautiful blooming plants available in a wide range of colors.

These are large beautiful flowers.

These have better shelf life and good market value.

They remove harmful airborne chemicals like formaldehyde, ammonia, toluene, xylene,

and allergens.

Due to Its importance of removing toxic substances from the air, NASA has placed it in

the list of air purifier plants. So option (a) is correct.

A SERIES

26

ADDITIONAL INFORMATION:

National Innovation Foundation-India (NIF) has recently boosted new varieties

of Anthurium, a flower with high market value.

These flowers which help to purify air are cultivated by a lady innovator from Kerala.

National Innovation Foundation-India

It is India’s national initiative to strengthen the grassroots technological

innovations and outstanding traditional knowledge.

Mission: To help India become a creative and knowledge-based society.

It was set up with the support of Honey Bee Network.

NIF supports grassroots innovations which are developed by individuals and local

communities in any technological field without any help from formal sector.

It also tries to ensure that such innovations spread widely through commercial

and/or non-commercial channels which will help in generating material or non-

material incentives for the innovators.

18. “The World Press Freedom Index” is annually released by which one of the

following organizations?

(a) World Economic Forum

(b) International Press Institute

(c) Reporters Without Borders

(d) World Press Institute

EXPLANATION:

The World Press Freedom Index is an annual ranking of countries compiled and

published by Reporters Without Borders. It is based upon the organization’s own

assessment of the countries’ press freedom records. It intends to reflect the degree of

freedom that journalists, news organisations, and netizens have in each country, and the

efforts made by authorities to respect this freedom. The report is partly based on a

questionnaire which asks questions about pluralism media independence, environment

and self-censorship, legislative framework, transparency, and infrastructure. So, option

(c) is correct.

A SERIES

27

ADDITIONAL INFORMATION:

World Press Freedom Index

India has dropped two places on a global press freedom index to be ranked 142nd

out of 180 countries in the annual World Press Freedom Report.

Norway is ranked first in the Index for the fourth year running.

India ranked better than its neighbours Pakistan (145) and Bangladesh (151), but

worse than Sri Lanka (127) and Nepal (112).

China at 177th position is just three places above North Korea, which is at 180th.

The report said that with no murders of journalists in India in 2019, as against six

in 2018.

However, there have been constant press freedom violations, including police

violence against journalists, ambushes by political activists, and reprisals instigated

by criminal groups or corrupt local officials.

According to the index the Middle East and North Africa region continues to be the

most difficult and dangerous for journalists.

Across the world, press freedom is under pressure from aggressive authoritarian

regimes.

The media is also facing a technological crisis, due to a lack of democratic

guarantees and a democratic crisis following polarization and repressive policies, the

report reads.

In addition comes a crisis of trust following growing suspicion and even hatred of

the media, and an economic crisis and impoverishing of quality journalism.

Among other issues, the report has listed coordinated social media hate campaigns

against journalists reporting on issues that “annoy right-wing followers”, criminal

prosecutions to gag journalists critical of authorities and police violence against

journalists.

SOURCE:

https://rsf.org/en/ranking

19. With reference to the Sufism, consider the following pairs:

Term Meaning

1. Khanqah - Hospice

2. Qawwals - Musicians

3. Ziyarat - Pilgrimage

A SERIES

28

Which of the pairs given above is/are correctly matched?

(a) 2 only

(b) 2 and 3 only

(c) 1 and 3 only

(d) 1, 2 and 3

EXPLANATION:

A khanqah or khaniqah is a building designed specifically for gatherings of a Sufi

brotherhood or tariqa and is a place for spiritual retreat and character reformation. In

the past, and to a lesser extent nowadays, they often served as hospices for saliks (Sufi

travelers), Murids (initiates) and talibs (Islamic students). Khanqahs are very often found

adjoined to dargahs (shrines of Sufi saints) and türbes (tombs of notables), mosques and

madrasas (Islamic schools).So, pair 1 is correct.

Qawwali is a musical vehicle by which a group of male musicians—called qawwals—

delivers inspirational Sufi messages to a traditionally male assembly of devotees. So, pair

2 is correct.

In Islamziyarat is a form of pilgrimage to sites associated with Muhammad, his family

members and descendants, his companions and other venerated figures in Islam such as

the prophets, Sufi auliya, and Islamic scholars. Sites of pilgrimage include mosques,

maqams, battlefields, mountains, and caves. So, pair 3 is correct.

20. Consider the following statements with reference to National Population Register

(NPR)

1. NPR database would contain demographic data and biometric particulars.

2. NPR database does not include a foreigner who is temporarily residing within

the political boundary of India for past 1 year.

3. Till now, NPR was once carried out in India only in the state of Assam.

Which of the statements given above is/are correct?

(a) 1 only

(b) 2 and 3 only

(c) 1 and 2 only

(d) 1, 2 and 3

EXPLANATION:

It collects both demographic data and biometric particulars. So statement 1 is correct.

It is a list of “usual residents of the country”.A “usual resident of the country” is one who

has been residing in a local area for at least the last six months, or intends to stay in a

particular location for the next six months.So statement 2 is not correct.

A SERIES

29

It is a nationwide project except for Assam. because in assam already NRC exercise has

been carried out. So statement 3 is not correct.

ADDITIONAL INFORMATION:

National Population Register

Definition:

It is a list of “usual residents of the country”.

A “usual resident of the country” is one who has been residing in a local area for at

least the last six months, or intends to stay in a particular location for the next six

months.

Legal Provisions:

The NPR is being prepared under provisions of the Citizenship Act 1955 and the

Citizenship (Registration of Citizens and Issue of National Identity Cards) Rules,

2003.

It is mandatory for every “usual resident of India” to register in the NPR.

Background:

The data for the NPR was first collected in 2010 along with the house listing phase

of Census 2011.

In 2015, this data was further updated by conducting a door-to-door survey.

However, with the use of Aadhaar as the key vehicle for transfer of government

benefits in the last few years, the NPR has taken a backseat.

Scope:

The NPR exercise is conducted at the local, sub-district, district, state and national

levels.

The NPR will collect both demographic data and biometric data. Biometric data will

be updated through Aadhar details.

In the 2010 exercise, the RGI had collected only demographic details.

In 2015, it updated the data further with the mobile, Aadhaar and ration card

numbers of residents.

For the 2020 exercise, it has dropped the ration card number but added other

categories.

A SERIES

30

Advantages:

It will streamline data of residents across various platforms.

For instance, it is common to find a different date of birth of a person on different

government documents. NPR will help eliminate that.

It will help the government formulate its policies better and also aid national

security.

It will help to target government beneficiaries in a better way and also further cut

down paperwork and red tape in a similar manner that Aadhaar has done.

It will help in implementing the idea of ‘One Identity Card’ that has been recently

floated by the government.

‘One Identity Card’ seeks to replace duplicate and siloed documentations of Aadhaar

card, voter ID card, banking card, passport, and more.

Privacy Concern:

There is no clarity on the mechanism for protection of the vast amount of data that

will be collected through NPR.

NPR and the NRC:

Unlike the NRC, the NPR is not a citizenship enumeration drive, as it would record

even a foreigner staying in a locality for more than six months.

With the government insisting that the NRC would be implemented across the

country, the NPR has raised anxieties around the idea of citizenship in the country.

All this is happening in the backdrop of the NRC in Assam which has excluded 19

lakh among the 3.3 crore who had applied.

NRC countrywide would only happen on the basis of the upcoming NPR.

After a list of residents is created (i.e. NPR), a nationwide NRC could go about

verifying the citizens from that list

SOURCE:

https://www.livemint.com/news/india/npr-vs-nrc-all-you-need-to-know-about-the-

national-population-register-11577186091362.html

21. Consider the following protected areas in India:

1. Sohola wetland

2. Dihing Patkai Elephant Reserve

A SERIES

31

3. Bura-Chapori Wildlife Sanctuary

4. Umananda Island

Which one of the following Indian state has all the above mentioned protected

areas?

(a) Meghalaya

(b) Assam

(c) Mizoram

(d) Manipur

EXPLANATION:

Sohola wetland, DihingPatkai Elephant Reserve, Bura-Chapori Wildlife Sanctuary,

Umananda Island are present in the state of assam. So option (b) is correct.

ADDITIONAL INFORMATION:

22. Which one of the following has the highest flow rate among the surface ocean

currents?

(a) Kuroshio current

(b) The Gulf stream

(c) Antarctic Circumpolar Current

(d) Agulhas Current

A SERIES

32

EXPLANATION:

Flow rate is defined as the volume of fluid flowing through an area each second.

Antarctic ocean current has the highest flow rate. So option (c) is correct.

ADDITIONAL INFORMATION:

The Antarctic Circumpolar Current, or ACC is changing as the world’s climate warms.

Scientists are studying the current to find out how it might affect the future of Antarctica’s

ice sheets, and the world’s sea levels.

It’s significance:

The Antarctic Circumpolar Current, or ACC, is the strongest ocean current on our planet. It

extends from the sea surface to the bottom of the ocean, and encircles Antarctica. It is vital

for Earth’s health because it keeps Antarctica cool and frozen.

The ACC carries an estimated 165 million to 182 million cubic metres of water every second

(a unit also called a “Sverdrup”) from west to east, more than 100 times the flow of all the

rivers on Earth. It provides the main connection between the Indian, Pacific and Atlantic

Oceans.

The tightest geographical constriction through which the current flows is Drake Passage,

where only 800 km separates South America from Antarctica. While elsewhere the ACC

appears to have a broad domain, it must also navigate steep undersea mountains that

constrain its path and steer it north and south across the Southern Ocean.

What is the Antarctic Circumpolar Current?

Antarctica is a frozen continent surrounded by icy waters. Moving northward, away from

Antarctica, the water temperatures rise slowly at first and then rapidly across a sharp

gradient. It is the ACC that maintains this boundary.

The ACC is created by the combined effects of strong westerly winds across the Southern

Ocean, and the big change in surface temperatures between the Equator and the poles.

Ocean density increases as water gets colder and as it gets more salty. The warm, salty

surface waters of the subtropics are much lighter than the cold, fresher waters close to

Antarctica. The depth of constant density levels slopes up towards Antarctica. The westerly

winds make this slope steeper, and the ACC rides eastward along it, faster where the slope

is steeper, and weaker where it’s flatter.

A SERIES

33

SOURCE:

https://www.indiatoday.in/education-today/gk-current-affairs/story/antarctic-

circumpolar-current-how-climate-change-is-affecting-the-world-s-most-influential-

ocean-current-1391669-2018-11-19

23. Consider the following statements about the working conditions of “Covid-19

Quarantine Alert System (CQAS)” which was developed by the Department of

Telematics in India:

1. It’s an app based service that works either with Wi-Fi or with cellular data.

2. It functions on the concept of location based service rendered completely by

the private cellular operators in India.

3. Operation with cellular data ensures the accuracy of location upto 2 Km.

Which of the statements given above is/are not correct?

(a) 3 only

(b) 1 and 2 only

(c) 1, 2 and 3 only

(d) None of the above

EXPLANATION:

Covid-19 Quarantine Alert System (CQAS) is an app based service that works either with

Wi-Fi or with cellular data.So, statement 1 is correct.

The CQAS prepares a list of mobile numbers, segregates them on the basis of telecom

service providers, and the location data provided by the telecom companies is run on the

application to create geo-fencing.So, statement 2 is correct.

This mobile application triggers e-mails and SMS alerts to an authorised government

agency if a person has jumped quarantine or escaped from isolation, based on the

person’s mobile phone’s cell tower location. The “geo-fencing” is accurate by up to 300

m.So, statement 3 is not correct.

ADDITIONAL INFORMATION:

The Department of Telecommunications (DoT) has shared a Standard Operating

Procedure (SOP) with all telecom service providers regarding the application called

COVID-19 Quarantine Alert System (CQAS).

A SERIES

34

CQAS collects phone data, including the device’s location, on a common secured

platform and alerts the local agencies in case of a violation by COVID patients under

watch or in isolation.

The CQAS prepares a list of mobile numbers, segregates them on the basis of

telecom service providers, and the location data provided by the telecom companies

is run on the application to create geo-fencing.

The location information is received periodically over a secure network for the

authorised cases with “due protection of the data received”.

The System triggers e-mails and SMS alerts to an authorised government agency if a

person has jumped quarantine or escaped from isolation, based on the person’s

mobile phone’s cell tower location. The “geo-fencing” is accurate by up to 300 m.

The States have been asked to seek the approval of their Home Secretaries under

the provisions of Section 5(2) of the Indian Telegraph Act, 1885, for the specified

mobile phone numbers to request the DoT to provide information by email or SMS in

case of violation of “geo-fencing”.

SOURCE:

https://www.thehindu.com/news/national/coronavirus-geo-fencing-app-will-be-used-

to-locate-quarantine-violators/article31241055.ece

24. Consider the following statements about the “Helicopter money” which was seen

in the news recently:

1. It involves printing large amounts of paper currency and distributing it to the

public.

2. It increases liquidity in the economic system and reduces the market interest

rates.

Which of the statements given above is/are correct?

(a) 1 only

(b) 2 only

(c) Both 1 and 2

(d) Neither 1 nor 2

EXPLANATION:

Helicopter Money involves printing large sums of money and distributing it to the

public.So, statement 1 is correct.

A SERIES

35

By using this Money, the liquidity will be increased among the public and the interest

rate will be brought down.So, statement 2 is correct.

ADDITIONAL INFORMATION:

Helicopter Money

This is an unconventional monetary policy tool aimed at bringing a flagging economy

back on track.

It involves printing large sums of money and distributing it to the public. American

economist Milton Friedman coined this term.

It basically denotes a helicopter dropping money from the sky.

Friedman used the term to signify “unexpectedly dumping money onto a struggling

economy with the intention to shock it out of a deep slump.”

Under such a policy, a central bank “directly increases the money supply and, via

the government, distributes the new cash to the population with the aim of boosting

demand and inflation.”

Helicopter money vs quantitative easing (QE)

Quantitative easing involves the use of printed money by central banks to buy

government bonds.

But not everyone views the money used in QE as helicopter money.

It sure means printing money to monetize government deficits, but the govt has to

pay back for the assets that the central bank buys.

It’s not the same as bond-buying by central banks “in which bank-owned assets are

swapped for new central bank reserves.

Helicopter money is also different from a central bank directly financing the debt of

a government.

SOURCE:

https://economictimes.indiatimes.com/news/et-explains/what-is-helicopter-money-

and-why-is-it-in-news/articleshow/75106564.cms

25. Which of the following farming practices help in Carbon Sequestration?

1. Mulching

2. Crop rotation

3. Strip cropping

A SERIES

36

Select the correct answer using the code given below.

(a) 1 and 2 only

(b) 1 and 3 only

(c) 2 and 3 only

(d) 1, 2 and 3

EXPLANATION:

Carbon sequestration or carbon dioxide removal (CDR) is the long-term removal, capture

or sequestration of carbon dioxide from the atmosphere to slow or reverse atmospheric

CO2 pollution and to mitigate or reverse global warming.Carbon dioxide (CO2) is

naturally captured from the atmosphere through biological, chemical, and physical

processes. These changes can be accelerated through changes in land use and

agricultural practices, such as converting crop and livestock grazing land into land for

non-crop fast growing plants.

A mulch is a layer of material applied to the surface of soil. Reasons for applying mulch

include conservation of soil moisture, improving fertility and health of the soil, reducing

weed growth and enhancing the visual appeal of the area.

Crop rotation is the practice of growing a series of different types of crops in the same

area across a sequenced of growing seasons. It reduces reliance on one set of nutrients,

pest and weed pressure, and the probability of developing resistant pest and weeds.

3. Strip cropping is a method of farming which involves cultivating a field partitioned into

long, narrow strips which are alternated in a crop rotation system. It is used when a

slope is too steep or when there is no alternative method of preventing soil erosion. All

three farming practices help in Carbon Sequestration. So, option (d) is correct.

SOURCE:

Environment by Shankar IAS academy

26. In India, the warnings for cyclones are given through the colours of Yellow, Orange

and Red. Which one of the following is the nodal agency for giving these warnings

in India?

(a) National Disaster Management Authority

(b) State Disaster Management Authority

(c) Indian Meteorological Department

(d) Indian National Centre for Ocean Information Services

EXPLANATION:

In India, the warnings for cyclones are given through the colours of Yellow, Orange and

A SERIES

37

Red.It is issued by the Indian Meteorological Department (IMD) whose objective is to alert

people ahead of severe or hazardous weather which has the potential to cause damage,

widespread disruption or danger to life.So, option (c) is correct.

ADDITIONAL INFORMATION:

Stages of Cyclone Warning

The cyclone warnings are issued to state government officials in four stages, they are

as follows

Pre cyclone watch - Issued 72 hours in advance contains early warning about the

development of a cyclonic disturbance in the north Indian Ocean, its likely

intensification into a tropical cyclone and the coastal belt likely to experience

adverse weather.

This early warning bulletin is issued by the Director General of Meteorology himself

and is addressed to the Cabinet Secretary and other senior officers of the

Government of India including the Chief Secretaries of concerned maritime states.

Cyclone Alert (Color code - Yellow) - It is issued at least 48 hrs. in advance of the

expected commencement of adverse weather over the coastal areas.

It contains information on the location and intensity of the storm likely direction of

its movement, intensification, coastal districts likely to experience adverse weather

and advice to fishermen, general public, media and disaster managers, This is

issued by the concerned ACWCs/CWCs and CWD at HQ.

Cyclone Warning (Color code - Orange) - It is issued at least 24 hours in advance

of the expected commencement of adverse weather over the coastal areas.

Landfall point is forecast at this stage. These warnings are issued by

ACWCs/CWCs/and CWD at HQ at 3 hourly interval giving the latest position of

cyclone and its intensity, likely point and time of landfall, associated heavy rainfall,

strong wind and storm surge along with their impact and advice to general public,

media, fishermen and disaster managers.

Post Landfall Outlook (Color code – Red) It is issued by the concerned

ACWCs/CWCs/and CWD at HQ at least 12 hours in advance of expected time of

landfall.

It gives likely direction of movement of the cyclone after its landfall and adverse

weather likely to be experienced in the interior areas.

A SERIES

38

27. Consider the following statements about “Tarabhai Shinde”:

1. She is the author of “Stree Purusha ki Tulana”.

2. She was associated with “Satyashodhak Samaj”.

3. She established an organisation called as “Arya Mahila Samaj”.

Which of the statements given above are correct?

(a) 1 and 2 only

(b) 2 and 3 only

(c) 1 and 3 only

(d) 1, 2 and 3

EXPLANATION:

'Stree-Purush Tulana' was written by Tarabai Shinde and was published in the year

1882. She was a feminist activist who protested patriarchy and caste in 19th century

India. This book was a comparison between women and men. It was very controversial

for its time in challenging the Hindu religious scriptures which acted as a source of

women oppression.So, statement 1 is correct.

Tarabhai Shindewas a member of Satyashodhak Samaj. Hence, she was associated with

Satyashodhak Samaj. So, statement 2 is correct.

Pandita Ramabai Sarasvatifounded Arya Mahila Samaj (Arya Women's Society). The

purpose of the society was to promote the cause of women's education and deliverance

from the oppression of child marriage. So, statement 3 is not correct.

ADDITIONAL INFORMATION:

Tarabai Shinde (1850–1910) was a feminist activist who protested patriarchy and

caste in 19th century India.

She is known for her published work, Stripurush Tulana ("A Comparison Between

Women and Men"), originally published in Marathi in 1882.

The pamphlet is a critique of upper-caste patriarchy, and is often considered the

first modern Indian feminist text. It was very controversial for its time in challenging

the Hindu religious scriptures themselves as a source of women's oppression, a view

that continues to be controversial and debated today.

Shinde was an associate of social activists Jotirao and Savitribai Phule and was a

founding member of their Satyashodhak Samaj ("Truth Finding Community")

organisation. The Phules shared with Shinde an awareness of the separate axes of

A SERIES

39

oppression that constitute gender and caste, as well as the intermeshed nature of

the two.

28. Who among the following are the members of National Human Rights Commission

(NHRC)?

1. Chairperson of National Commission for Scheduled Castes

2. Chairperson of National Commission for Backward Classes

3. Chief Commissioner for Persons with Disabilities

4. Chairperson of National Commission for Women

5. Chairperson of National Commission for the Protection of Child Rights

Select the correct answer using the code given below.

(a) 1, 2 and 4 only

(b) 2, 3 and 5 only

(c) 1, 3, 4 and 5 only

(d) 1, 2, 3, 4 and 5

EXPLANATION:

Composition of NHRC: The Bill provides that a person who has been Chief Justice

of the Supreme Court, or a Judge of the Supreme Court can be the chairperson of

the NHRC.

The Bill allows 3 members to be appointed, of which at least one will be a woman.

Members of the NHRC: The chairpersons of the National Commission for

Scheduled Castes, National Commission for Scheduled Tribes, and National

Commission for Women, National Commission for Backward Classes, the National

Commission for the Protection of Child Rights, and the Chief Commissioner for

Persons with Disabilities.

Chairperson of SHRC: The Bill provides for a person who has been Chief Justice

or Judge of a High Court can be chairperson of a SHRC.

So option (b) is correct.

ADDITIONAL INFORMATION:

What are the provisions of the Act?

Composition of NHRC: The chairperson of the NHRC is a person who has been a

Chief Justice of the Supreme Court.

The Act provides for 2 persons having knowledge of human rights to be appointed as

members of the NHRC.

A SERIES

40

Members of the NHRC: The chairpersons of the National Commission for Scheduled

Castes, National Commission for Scheduled Tribes, and National Commission for

Women.

Chairperson of SHRC: A person who has been a Chief Justice of a High Court.

Term of office: The chairperson and members of the NHRC and SHRC will hold office

for 5 years or till the age of 70 years, whichever is earlier.

The Act allows for the reappointment of members of the NHRC and SHRCs for a

period of 5 years.

Powers of Secretary-General: The Act provides for a Secretary-General of the NHRC

and a Secretary of a SHRC, who exercise powers as may be delegated to them.

What are the amendments made?

Composition of NHRC: The Bill provides that a person who has been Chief Justice of

the Supreme Court, or a Judge of the Supreme Court can be the chairperson of the

NHRC.

The Bill allows 3 members to be appointed, of which at least one will be a woman.

Members of the NHRC: The chairpersons of the National Commission for Scheduled

Castes, National Commission for Scheduled Tribes, and National Commission for

Women, National Commission for Backward Classes, the National Commission for

the Protection of Child Rights, and the Chief Commissioner for Persons with

Disabilities.

Chairperson of SHRC: The Bill provides for a person who has been Chief Justice or

Judge of a High Court can be chairperson of a SHRC.

Term of office: The Bill reduces the term of office to 3 years or till the age of 70

years, whichever is earlier.

The Bill removes the five-year limit for reappointment found in the act.

Powers of Secretary-General: The Bill allows the Secretary-General and Secretary to

exercise all administrative and financial powers (except judicial functions), subject

to the respective chairperson’s control.

Union Territories: The Bill provides that the central government may confer on a

SHRC human rights functions being discharged by Union Territories. Functions

relating to human rights in the case of Delhi will be dealt with by the NHRC.

SOURCE:

https://www.iasparliament.com/current-affairs/the-protection-of-human-rights-

amendment-bill-

2019#:~:text=The%20Protection%20of%20Human%20Rights%20(Amendment)%20Bill

%2C%202019%20was,of%20Human%20Rights%20Act%2C%201993.

A SERIES

41

29. Consider the following statements about the Wildlife Protection Act, 1972:

1. Scheduled Tribes in Andaman and Nicobar Islands cannot hunt any animals

protected under the Schedules I and II of this act.

2. It bans international trading of any animals protected under Schedule I and II

of this act.

Which of the statements given above is/are correct?

(a) 1 only

(b) 2 only

(c) Both 1 and 2

(d) Neither 1 nor 2

EXPLANATION:

Wildlife ProtectionAct, 1972 does not affect the hunting rights of the Scheduled Tribes in

Andaman and Nicobar Islands. So, statement 1 is not correct.

It has provisions for the Convention on International Trade in Endangered Species of

Wild Fauna and Flora (CITES). It bans only illegal trade (not legal trade). So, statement

2 is not correct.

ADDITIONAL INFORMATION:

The Wildlife (Protection) Act of 1972

The passing of the Wildlife Act of 1972 constitutes an important landmark in the

history of wildlife legislationin the country.

This is because of the fact that the “Forest” including “Wildlife” was then a State

subject falling in Entry 20 List II of Seventh Schedule, Parliament had no power to

make law on the same except as provided in Articles249,250 and 252 of the

constitution.

Having regard to the importance of the matter, the Act has been adopted by all the

States except that of Jammu and Kashmir which has a similar law enacted for the

purpose of wildlife protection. The operation of the Actis mandatory in the Union

Territories too.

The Wildlife (Protection) Act of 1972 provides the basic framework to ensure the

protection and management of wildlife. The Act was amended subsequently in 1982,

1986, 1991 and 1993 to accommodate provision for itseffective implementation.

Salient features of the Act:

A SERIES

42

The Wildlife Protection Act, 1972 is a product of process which started long ago in

1887 for the protection of a few wild birds and after addition of wild animals in 1912

and specified plants in 1919 it covered almost all the wildlife resources which need

protection and management.

The rating of the Schedules I to V is in accordance with the risk of survival of the

wildlife (fauna) enlisted in them. Animals included Schedule are provided for total

protection from hunting and the trade and commerce related to such animals are

strictly regulated. The schedule VI has been added to include the specified plant

species to be protected by the Wildlife (Protection)Amendment Act of 1991.

An expert committee, constituted by the Indian Board of Wildlife considers

amendments to the Act, as andwhen necessary.

With the amendment of the Act in 1991, powers of the State Governments have been

withdrawn almost totally. Now the State Governments are not empowered to declare

any wild animal a vermin. Further by addition of provision, immunization of

livestock within a radius of 5 km from a National Park or sanctuary hasbeen made

compulsory.

SOURCE:

Environment by Shankar IAS academy

30. Consider the following statements about a country:

1. Its coastline has a warm ocean current.

2. It experiences summer in the month of December.

3. The country has a Human Development Index of approximately 0.75.

Based on the above statements, the country is

(a) Mexico

(b) The Bahamas

(c) Australia

(d) Brazil

A SERIES

43

EXPLANATION:

Both south equatorial and brazil current are warm current. It has HDI of 0.75. As major

part of brazil is in southern hemisphere it experiences summer in December. So option

(d) is correct.

31. With reference to medieval history of India, which of the following schools of

miniature paintings originated in the Himalayan foothills around the modern-day

state of Himachal Pradesh?

1. Basohli

2. Bundi

3. Kangra

Select the correct answer using the code given below.

(a) 2 only

(b) 1 and 3 only

(c) 2 and 3 only

(d) 1, 2 and 3

EXPLANATION:

Basohli painting was a vigorous, bold and imaginative artistic style, rich, stylish and

unconventional. A style of painting characterized by vigorous use of primary colours and

a peculiar facial formula prevailed in the seventeenth and early eighteenth centuries in

A SERIES

44

the foothills of the Western Himalayas in the Jammu and Punjab States. The earliest

paintings in this style have been dated to the time of Raja Kirpal Pal.

Bundi painting is an important school of the Rajasthani style of Indian miniature

painting that originated in the princely state of Bundi, located between Jaipur and

Udaipur. The blend of Mughal and Deccani art elements in Bundi style are unique. Bundi

paintings excelled in its illustrations of Krishna-Lila and Rasikapriya.

Kangra painting is the pictorial art of Kangra, named after Kangra, Himachal Pradesh, a

former princely state, which patronized the art. It became prevalent with the fading of

Basohli school of painting in mid-18th century, and soon produced such a magnitude in

paintings both in content as well as volume, that the Pahari painting school, came to be

known as Kangra paintings. So, option (b) is correct.

32. In India, a particular industry has been declared as a “Public Utility Service”

under which one of the following Acts?

(a) Industrial Disputes Act, 1947

(b) Minimum Wages Act, 1948

(c) Trade Unions Act, 1926

(d) Factories Act, 1948

EXPLANATION:

The Union Government has declared the banking industry as a public utility service for

six months till October 21 under the provisions of the Industrial Disputes Act. The move

means that the banking sector would not see any strikes by employees or officers during

the operation of the act starting from April 21. The Department of Financial Services

announced that Labour and Employment Ministry has declared the banking industry as

a public utility service for six months till October 21. So, option (a) is correct.

ADDITIONAL INFORMATION:

The Industrial Disputes Act, 1947 extends to the whole of India and regulates Indian

labour law so far as that concerns trade unions as well as Individual workman

employed in any Industry within the territory of Indian mainland. Enacted on 11th

March 1947 and It came into force 1 April 1947.

An act to make provision for the investigation and settlement of industrial disputes,

and for certain other purposes.

The objective of the Industrial Disputes Act is to secure industrial peace and

harmony by providing mechanism and procedure for the investigation and

A SERIES

45

settlement of industrial disputes by conciliation, arbitration and adjudication which

is provided under the statute.

The main and ultimate objective of this act is "Maintenance of Peaceful work culture

in the Industry in India" which is clearly provided under the Statement of Objects &

Reasons of the statute.

The laws apply only to the organised sector. Chapter V talks about the most

important and often in news topic of 'Strikes and Lockouts'. It talks about the

Regulation of strikes and lockouts and the proper procedure which is to be followed

to make it a Legal instrument of 'Economic Coercion' either by the Employer or by

the Workmen. Chapter V-B, introduced by an amendment in 1976, requires firms

employing 300 or more workers to obtain government permission for layoffs,

retrenchments and closures. A further amendment in 1982 (which took effect in

1984) expanded its ambit by reducing the threshold to 100 workers.

The Act also lays down:

o The provision for payment of compensation to the workman on account of

closure or lay off or retrenchment.

o The procedure for prior permission of appropriate Government for laying off

or retrenching the workers or closing down industrial establishments

o Unfair labour practices on part of an employer or a trade union or workers.

SOURCE:

https://www.hindustantimes.com/business-news/banking-declared-as-public-utility-

service-for-six-months/story-

iwKMcxLxO4kIzzUI7ARWbL.html#:~:text=The%20government%20has%20declared%2

0banking,of%20the%20Industrial%20Disputes%20Act.&text=All%20public%20sector

%20banks%2C%20old,Bank%20are%20members%20of%20IBA.

33. Which one of the following National Park is a part of Biosphere Reserve, UNESCO

World Heritage Site and UNESCO World network of Biosphere Reserve?

(a) Nanda Devi National Park

(b) Corbett National Park

(c) Nokrek National Park

(d) Khangchendzonga National Park

EXPLANATION:

Nanda devi national park comes under Biosphere reserve, UNESCO World heritage site

A SERIES

46

and also UNESCO World network of biosphere reserve.So option (a) is correct.

Corbett national park is not a UNESCO world heritage site. So option (b) is not correct.

Nokrek is a biosphere reserve.So option (c) is not correct.

Khangchendzonga National Park is UNESCO world network of biosphere reserve and

biosphere reserve. So option (d) is not correct.

ADDITIONAL INFORMATION:

Unesco world heritage sites

Kaziranga National Park Assam

Keoladeo Ghana National Park Rajasthan

Manas Wildlife Sanctuary Assam

Nanda Devi National Park and Valley of Flowers Uttarakhand

Sundarbans National Park West Bengal

Western Ghats Maharashtra,

Goa,

Karnataka,

Tamil Nadu and

Kerala

Great Himalayan National Park Himachal Pradesh

Unesco world network of biosphere reserve

# Name

1 Nilgiri Biosphere Reserve

2 Gulf of Mannar Biosphere Reserve

A SERIES

47

# Name

3 Sundarbans Biosphere Reserve

4 Nanda Devi Biosphere Reserve

5 Nokrek Biosphere Reserve

6 Pachmarhi Biosphere Reserve

7 Simlipal Biosphere Reserve

8 Great Nicobar Biosphere Reserve

9 Achanakmar-Amarkantak Biosphere Reserve

10 Agasthyamalai Biosphere Reserve

11 Khangchendzonga National Park

Biosphere reserve in india

A SERIES

48

34. Consider the following statements about the “Swamitva scheme”, which was

recently launched by the Government of India:

1. This scheme will create records of land ownership in villages and would help in

reducing the disputes over property.

2. It was launched on a pilot basis for the integrated development of 1000

villages each with more than 50% Scheduled Caste population.

Which of the statements given above is/are correct?

(a) 1 only

(b) 2 only

(c) Both 1 and 2

(d) Neither 1 nor 2

EXPLANATION:

The Swamitva scheme will create records of land ownership in villages and would help in

reducing the disputes over property. So, statement 1 is correct.

Pradhan Mantri Adarsh Gram Yojana (PMAGY), a Government of India initiative for the

empowerment of deprived sections, aims to achieve integrated development of selected

villages through the convergent implementation of all relevant Central and State

schemes. The scheme was launched in March 2010 on a pilot basis for the integrated

development of 1000 villages, each with more than 50% SC population. So, statement 2

is not correct.

ADDITIONAL INFORMATION:

Swamitva Yojana is meant to create a record of land ownership in rural areas using

modern technology.

The scheme is piloted by the Panchayati Raj ministry of the Union government and

has been launched on April 24 2020, the Panchayati Raj Diwas.

The need for this Yojana was felt since several villagers in the rural areas don’t have

papers proving ownership of their land. In most states, survey and measurement of

the populated areas in the villages has not been done for the purpose of

attestation/verification of properties.

Swamitva Yojana is aimed to fill the above gap to provide ownership rights to people

in the villages. It is expected to go a long way in settling property rights in rural

hinterlands and likely to become a tool for empowerment and entitlement, reducing

social strife on account of discord over properties.

A SERIES

49

The residential land in villages will be measured using drones to create a non-

disputable record. It is the latest technology for surveying and measuring of land.

The scheme will be carried out in close coordination with the Central Panchayati Raj

ministry, Survey of India, Panchayati Raj departments and Revenue departments of

Drones will draw a digital map of every property falling within the geographical

limits of a village and demarcate the boundaries of every revenue area.

Property card for every property in the village will be prepared by states using

accurate measurements delivered by drone-mapping. These cards will be given to

property owners and will be recognised by the land revenue records department.

The delivery of property rights through an official document will enable villagers to

access bank finance using their property as collateral.

The property records for a village will also be maintained at the Panchayat level,

allowing for the collection of associated taxes from the owners. The money generated

from these local taxes will be used to build rural infrastructure and facilities.

Freeing the residential properties including land of title disputes and the creation of

an official record is likely to result in appreciation in the market value of the

properties.

The accurate property records can be used for facilitating tax collection, new

building and structure plan, issuing of permits and for thwarting attempts at

property grabbing

SOURCE:

https://www.hindustantimes.com/india-news/what-is-swamitva-yojana-launched-by-

pm-narendra-modi-all-you-need-to-know/story-

UpshkST02eW59ZGsF9jxLK.html#:~:text=1.,2020%2C%20the%20Panchayati%20Raj%

20Diwas.

35. For the same insolation, continents heat up faster when compared to the oceans

because

(a) Land is a very good conductor of heat

(b) Water has a high specific heat capacity

(c) Ocean topography is much more complex than land

(d) Plants absorb large amount of heat

EXPLANATION:

The specific heat is the amount of heat per unit mass required to raise the temperature

by one degree Celsius. It is high for water so water needs more heat to show high

A SERIES

50

temperature. So continents heat up faster compared to the oceans for the Same

insolation. So option (b) is correct.

Source:

Physical Geography NCERT

36. The Common Services Centres (CSC) is a strategic cornerstone of the National e-

Governance Plan (NeGP), to introduce e-governance on a massive scale. Which of

the following kind of services are offered under this CSC scheme?

1. Rural Banking and Insurance Services including Micro-credit.

2. Commercial Services including Internet Browsing and Village level BPO.

3. Agriculture Services including Sericulture, Animal Husbandry and Fisheries.

4. Health Services including Telemedicine and Health Check-ups.

Select the correct answer using the code given below.

(a) 1 and 2 only

(b) 1, 2 and 3 only

(c) 1, 2 and 4 only

(d) 1, 2, 3 and 4

EXPLANATION:

The following kind of services are offered under this CSC scheme are

Agriculture Services (Agriculture, Horticulture, Sericulture, Animal Husbandry,

Fisheries, Veterinary)

Education & Training Services (School, College, Vocational Education, Employment,

etc.)

Health Services (Telemedicine, Health Check-ups, Medicines)

Rural Banking & Insurance Services (Micro-credit, Loans, Insurance)

Entertainment Services (Movies, Television)

Utility Services (Bill Payments, Online bookings)

Commercial Services (DTP, Printing, Internet Browsing, Village level BPO).

So, option (d) is correct.

A SERIES

51

ADDITIONAL INFORMATION:

Common Services Centres

The CSC is a strategic cornerstone of the National e-Governance Plan (NeGP),

approved by the Government in May 2006, as part of its commitment in the National

Common Minimum Programme to introduce e-governance on a massive scale.

The CSCs would provide high quality and cost-effective video, voice and data content

and services, in the areas of e-governance, education, health, telemedicine,

entertainment as well as other private services.

A highlight of the CSCs is that it will offer web-enabled e-governance services in

rural areas, including application forms, certificates, and utility payments such as

electricity, telephone and water bills

The Scheme creates a conducive environment for the private sector and NGOs to

play an active role in implementation of the CSC Scheme, thereby becoming a

partner of the government in development of rural India.

The PPP model of the CSC scheme envisages a 3-tier structure consisting of the CSC

operator (called Village Level Entrepreneur or VLE); the Service Centre Agency (SCA),

that will be responsible for a division of 500-1000 CSCs; and a State Designated

Agency (SDA) identified by the State Government responsible for managing the

implementation in the entire State.

SOURCE:

https://meity.gov.in/content/csc-

scheme#:~:text=Agriculture%20Services%20(Agriculture%2C%20Horticulture%2C,%2

Dcredit%2C%20Loans%2C%20Insurance)

37. Consider the following statements about the Market Intervention Scheme (MIS) in

India:

1. It is a price support mechanism for procurement of raw cotton, raw jute and

copra in the event of a fall in market prices.

2. The Food Corporation of India is responsible for implementing this scheme and

procures the commodities at a fixed Market Intervention Price.

Which of the statements given above is/are correct?

(a) 1 only

(b) 2 only

(c) Both 1 and 2

(d) Neither 1 nor 2

A SERIES

52

EXPLANATION:

MIS is a price support mechanism implemented on the request of State Governments for

the procurement of perishable and horticultural commodities in the event of a fall in

market prices.So, statement 1 is not correct.

The Department of Agriculture & Cooperation is implementing the scheme.So,

statement 2 is not correct.

ADDITIONAL INFORMATION:

Market Intervention Scheme

MIS is a price support mechanism implemented on the request of State

Governments for the procurement of perishable and horticultural commodities in

the event of a fall in market prices.

It is implemented when there is at least a 10% increase in production or a 10%

decrease in the ruling rates over the previous normal year.

MIS works in a similar fashion to Minimum Support Price based procurement

mechanism for food grains but is an ad-hoc mechanism.

Its objective is to protect the growers of these horticultural/agricultural commodities

from making distress sale in the event of the bumper crop.

Under MIS, support can be provided in some years, for a limited but defined period,

in specified critical markets and by purchasing specified quantities. The initiative

has to emerge from the concerned state.

Commodities covered

The MIS has been implemented in case of commodities like apples, garlic, oranges,

grapes, mushrooms, clove, black pepper, pineapple, ginger, red-chillies, coriander

seed, chicory, onions, potatoes, cabbage, mustard seed, castor seed, copra, palm oil

etc.

Remuneration under MIS

MIS provides remunerative prices to the farmers in case of glut in production and

fall in prices.

Proposal of MIS is approved on the specific request of State/UT Government, if they

are ready to bear 50% loss (25% in case of North-Eastern States), if any, incurred on

its implementation.

A SERIES

53

Further, the extent of total amount of loss shared is restricted to 25% of the total

procurement value which includes cost of the commodity procured plus permitted

overhead expenses.

SOURCE:

http://www.arthapedia.in/index.php?title=Market_Intervention_Scheme_(MIS)

38. Consider the following statements about the Parliamentary privileges in India:

1. President of India, who is integral part of the Indian Parliament, also enjoys

these privileges.

2. Normally, a Member of Parliament can raise a question involving a breach of

privilege in the Houses of Parliament even without the consent of the

Presiding Officer of that House.

3. The Parliament of India has not codified its privileges.

Which of the statements given above is/are correct?

(a) 3 only

(b) 2 and 3 only

(c) 1 and 2 only

(d) 1, 2 and 3

EXPLANATION:

President, though an integral part of the parliament, does not enjoy parlimenry privileges.

These rights are mainly from the members of both the Houses of Parliament. Apart from

this, these rights are also given to those individuals who speak and participate in any

committee of the Parliament, which includes the Attorney General of India and the Union

Ministers.

So statement 1 is not correct.

A member can raise a question involving a breach of privilege only with the consent of the

Speaker or the Chairperson. So statement 2 is not correct.

The Parliament has not yet codified its privileges. So statement 3 is correct.

ADDITIONAL INFORMATION:

Originally the constitution envisaged two types of privileges under the article 105 of the

Indian constitution. One is freedom of speech in Parliament and the right of publication of

its proceedings.

A SERIES

54

Concept of parliamentary privilege in the Constitution of India has been taken from the

British Constitution. The main motive of these privileges is to uphold the supremacy of the

office of the Parliament and its members.

What are Parliamentary Privileges?

Parliamentary privileges are special rights, immunities, exceptions enjoyed by the members

of the two houses of Parliament and their committees.

Who Enjoys Parliamentary Privileges

These rights are mainly from the members of both the Houses of Parliament. Apart from

this, these rights are also given to those individuals who speak and participate in any

committee of the Parliament, which includes the Attorney General of India and the Union

Ministers.

It is necessary to tell that the President, even if he is part of Parliament, does not have

parliamentary privileges.

Parliamentary privileges can be broadly divided into two categories;

A. Enjoyed by the Members of Parliament collectively

B. Enjoyed by the Members of Parliament individually

The collective privileges of Members of Parliament are as follows;

1. No person (either a member or outsider) can be arrested and no legal process (criminal or

civil) can be initiated within the premises of the house without the permission of the

presiding officer of the house.

2. No Court has the right to investigate proceedings of the House or any of its committees.

3. Parliament can exclude guests from its proceedings and in some cases of national

interest it can also hold a secret meeting on any important matter.

4. Parliament can punish members as well as outsiders for breach of its privileges or its

contempt by reprimand, admonition or imprisonment (also suspension or expulsion in case

of members).

Individual Privileges are as Follows;

1. When the Parliament is in session, a Member of Parliament or a privileged person may

refuse to appear in court or to present any evidence in a court.

2. The members of Parliament can’t be arrested during the session of the Parliament and

40 days before the beginning and 40 days after the end of the session. However this

privilege is available in civil cases only not in criminal cases.

A SERIES

55

3. No member is liable to any proceedings in any given court for anything said or any vote

by him/her in the parliament or its committees.

What is called breach of Parliamentary Privileges;

If any person or officer violates the individual or collective privileges of a Member of

Parliament, such as, disrespect, abuse, attack, etc., such acts are considered as breach of

Parliamentary Privileges which is punishable by the house.

Thus, the above facts make it clear that Parliamentary Privileges in India have been

enforced so that the respect of Parliament its members can be ensured.

But it is observed that after winning the election, these representatives do not respect the

public while on the other hand they expect that public should give them due respect.

Apart from this, you may have noticed that Parliamentarians are often seen misbehaving

with the officials and the general public. This is called misuse of parliamentary privileges.

Therefore, it is the need of the hour that if government want to maintain the dignity of the

democracy then it should make appropriate changes in the parliamentary privileges

because nobody can be superior to the general public.

SOURCE:

Indian Polity - Laxmikanth

39. Consider the following passage about a river in India:

“It is a tributary of the river Godavari. It passes through the states of

Maharashtra, Karnataka and Telangana. It originates in the Balaghat range of hills

and flows on the Balaghat plateau along with its tributaries - Terna, Tawarja and

Gharni. Nizam Sagar project is constructed on this river.

Which one of the following river is described in the above passage?

(a) Krishna

(b) Bhima

(c) Pranhita

(d) Manjira

EXPLANATION:

The river mentioned above is manjira.So option (d) is correct

A SERIES

56

ADDITIONAL INFORMATION:

NizamSagar Dam is an Indian dam named after the Nizam of Hyderabad. It is

a reservoir constructed across the Manjira River, a tributary of the Godavari

River, between Achampet and BanjePally villages of the Kamareddy

district in Telangana, India. It is located at about 144 km north-west of Hyderabad.

NizamSagar is the oldest dam in the state of Telangana.

SOURCE:

https://www.newindianexpress.com/states/telangana/2019/aug/11/andhra-pradesh-

and-telangana-decide-to-release-nagarjuna-sagar-project-water-for-farmers-

2017274.html

40. Geofencing technology can be great if you are a retailer operating on a local scale.

There are several ways to enhance the business by targeting the customers on

super granular level. Which of the following applications uses the Geofencing

technology?

1. Social networking filters

2. Marketing promotions

3. Monitoring Assets

4. Turnaround times in logistics

Select the correct answer using the code given below.

(a) 1 and 3 only

(b) 2 and 4 only

(c) 2, 3 and 4 only

(d) 1, 2, 3 and 4

EXPLANATION:

The common geofencing applications are

Social networking: One of the most recognizable uses for geofencing comes in the

form of popular social networking apps — most notably, Snapchat. Location-

based filters, stickers and other shareable content are all made possible with

geofencing. Whether you’re using a promoted filter at a concert, using a custom-

made filter for a friend’s birthday or uploading to public, location-based stories,

it’s all thanks to these virtual perimeters.

A SERIES

57

Marketing: Besides social networking, geofencing is also a popular way for

businesses to deliver in-store promotions, alerting you right as you step in range

of the store. Geofencing also helps businesses target ads to a specific audience to

figure out what strategies work best based off user’s location-data.

Audience engagement: Geofencing is used to engage crowds of people at organized

events, like concerts, festivals, fairs and more. For example, a concert venue

might use a geofence to crowdsource social media posts or deliver information

about the venue or event.

Smart appliances: As more of our appliances get “smart,” with Bluetooth

capabilities, it’s easier than ever to program your fridge to remind you that you’re

out of milk the next time you pass by the grocery store. Or you can make sure the

thermostat is set to the perfect temperature when you get home from work by

using a geofence.

Human resources: Some companies rely on geofencing for monitoring employees,

especially workers who spend time off-site doing field work. It’s also an easy way

to automate time cards, clocking employees in and out as they come and go.

Telematics: Geofencing can also be helpful with telematics, allowing companies to

draw virtual zones around sites, work areas and secure areas. They can be

triggered by a vehicle or a person and send alerts or warnings to the operator.

Security: Geofencing might seem invasive — and it certainly has the potential to

sometimes feel like an overreach depending on how it’s used. However, geofencing

can also be used to bring more security to your mobile device. For example, you

can set your phone to unlock when you’re home using a geofence or to get alerts

when someone enters the house or leaves.

Monitoring assets - Not only it helps in tracking assets, but it is also possible to

check on the frequency of a particular asset. It can be a machine that works in a

fixed location or a vehicle that is running down the road. It is possible to read all

the data when that particular asset falls in the area.

Turnaround times - Geofencing technology can be used to monitor asset timing –

entry and exit – from a specific location. Through this, it is possible to track down

the turnaround time and also to identify whether the location performance is poor

or accurate. This sort of application is usually followed in the logistics industry or

transport sector to keep a track on the vehicles. It is easy to check that the

products are loaded, unloaded or reloaded on time to reach the end point.

So, option (d) is correct.

A SERIES

58

ADDITIONAL INFORMATION:

GeoFencing is a technology that helps in determining geographical boundaries. It is

used for GPS and RFID (Radio Frequency Identification).

This technology allows anyone to set up triggers based on location information.

When any person or device enters a specific boundary or area, then it will issue an

alert. In simple words, it is a virtual map perimeter through which it is easy to track

in real-time.

It is most commonly used as a perimeter for a particular point where a diameter can

be a house, a store or can be spread out to some miles.

However, there are many other uses of geofencing, for instance Google Earth which

manages it with the help of satellites that displays the boundaries of a particular

area. On another hand, it is used as per the latitude and longitudinal boundaries of

the user and web-based maps.

Types of GeoFencing

There are two types of geofence virtual barriers.

o Active geofence depends on the end-user – it can easily use location services

to track them down. It can also be done with the help of a certain

smartphone apps.

o Passive geofencing is one which stays on all the time. These type of fencing

depends on internet connectivity that can be cellular or via Wi-Fi rather than

RFID and GPS. They always work in the background if the location on a

smartphone is switched on.

41. With reference to the Sikhism, who among the following laid the foundation of the

“Khalsa Panth” and defined its five symbols?

(a) Guru Nanak

(b) Guru Arjan Dev

(c) Guru Tegh Bahadur

(d) Guru Gobind Singh

EXPLANATION:

Khalsa refers to both a community that considers Sikhism as its faith, as well as a

special group of initiated Sikhs. The Khalsa tradition was initiated in 1699 by the last

living Guru of Sikhism, Guru Gobind Singh. Its formation was a key event in the history

of Sikhism. The founding of Khalsa is celebrated by Sikhs during the festival of Vaisakhi.

A SERIES

59

Guru Gobind Singh created and initiated the Khalsa as a warrior with a duty to protect

the innocent from any form of religious persecution.The Khalsa redefined the Sikh

tradition from the start. It formulated an initiation ceremony (amrit pahul, nectar

ceremony) and rules of conduct for the Khalsa warriors. It created a new institution for

the temporal leadership of the Sikhs, replacing the masand system maintained by the

earlier Gurus of Sikhism. Additionally, the Khalsa provided a political and religious vision

for the Sikh community.

Upon initiation, a Khalsa Sikh was given the titles of Singh (male) meaning lion and Kaur

(female) meaning princess. The rules of life, included behavioral code (Rahit, such as no

tobacco, no alcohol, no adultery, no halal meat), and a dress code (Five Ks). They are:

Kesh (uncut hair), Kangha (a wooden comb for the hair), Kara (an iron bracelet), Kachera

(100% cotton tieable undergarment) (not an elastic one) and Kirpan (an iron dagger large

enough to defend oneself).So, option (d) is correct.

ADDITIONAL INFORMATION:

Sikh Gurus

The era of the ten gurus of Sikhism spans from the birth of Nanak Dev in 1469,

through the life of Guru Gobind Singh.

At the time of Guru Gobind Singh’s death in 1708, he passed the title of Guru to the

Sikh scripture, Guru Granth.

1. Guru Nanak Dev - Guru from 1469 to 1539

Guru Nanak Dev, first of the 10 gurus, founded the Sikh faith, introducing the

concept of one God.

He started the institution of Guru Ka Langar. Langar is the term in the Sikh religion

refers to the common kitchen where food is served to everyone without any

discrimination.

He emphasized the equality of women and rejected the path of renunciation and he

rejected the authority of the Vedas.

He was the contemporary of Mughal emperor - Babur.

2. Guru Angad Dev - Guru from 1539 to 1552

Guru Angad Dev, second of the 10 gurus, invented and introduced the Gurmukhi

(written form of Punjabi) script.

He compiled the writings of Nanak Dev in Guru Granth Sahib in Gurmukhi Script.

A SERIES

60

Popularized and expanded the institution of Guru ka Langar which was started by

Guru Nanak Dev.

3. Guru Amardas Sahib - Guru from 1552 to 1574

Guru Amardas introduced the Anand Karaj marriage ceremony for the Sikhs,

replacing the Hindu form.

He established Manji & Piri system of religious missions for men and women

respectively.

He strengthened the tradition of Guru Ka Langar.

He also completely abolished amongst the Sikhs, the custom of Sati and purdah

system.

He was the contemporary of Mughal emperor - Akbar.

4. Guru Ram Das - Guru from 1574 to 1581

Guru Ram Das, fourth of the 10 gurus, founded the city of Amritsar.

He started the construction of the famous Golden Temple at Amritsar, the holy city

of the Sikhs.

He requested the Muslim Sufi, Mian Mir to lay the cornerstone of the Harmandir

Sahib.

5. Guru Arjan Dev - Guru from 1581 to 1606

He compiled the Adi Granth, the scriptures of the Sikhs.

He completed construction of Sri Darbar Sahib also known as Golden Temple in

Amritsar.

He founded the town of Tarn Taran Sahib near Goindwal Sahib.

He became the first great martyr in Sikh history when Emperor Jahangir ordered his

execution. Thus, he was hailed as Shaheedan-de-Sartaj (The crown of martyrs).

6. Guru Har Gobind Sahib - Guru from 1606 to 1644

He was the son of Guru Arjan Dev and was known as a "soldier saint”.

He organised a small army and became the first Guru to take up arms to defend the

faith.

He waged wars against Mughal rulers Jahangir and Shah Jahan.

7. Guru Har Rai Sahib - Guru from 1644 to 1661

Though he was a man of peace, he never disbanded the armed sikh warriors who

were earlier maintained by Guru Har Gobind.

A SERIES

61

He gave shelter to Dara Shikoh, the eldest son of Mughal Ruler Shah Jahan, who

was later persecuted by Aurangazeb.

He cautiously avoided conflict with Emperor Aurangzeb and devoted his efforts to

missionary work.

8. Guru Har Krishan Sahib - Guru from 1661 to 1664

Guru Har Krishan was the youngest of the Gurus. He was installed as Guru at the

age of five.

He was contemporary of Aurangazeb and summoned to Delhi by him under framed

charges of anti-Islamic blasphemy.

9. Guru Tegh Bahadur Sahib - Guru from 1665 to 1675

He established the town of Anandpur.

He opposed the forced conversion of the Hindu Kashmiri Pandits by Mughal ruler

Aurangazeb and he was consequently persecuted for this.

10. Guru Gobind Singh Sahib - Guru from 1675 to 1708

He became Guru after the martyrdom of his father Guru Tegh Bahadur.

He created the Khalsa in 1699, changing the Sikhs into a saint-soldier order for

protecting themselves.

Last Sikh Guru in human form and he passed the Guruship of the Sikhs to the

Guru Granth Sahib.

Guru Granth Sahib

Guru Granth Sahib (also known as the Adi Granth) is the scripture of the Sikhs.

The Granth was written in Gurmukhi script and it contains the actual words and

verses as uttered by the Sikh Gurus.

It is considered the Supreme Spiritual Authority and Head of the Sikh religion,

rather than any living person.

42. In the context of the economy, which one of the following indicator is used to

measure whether people in one country is better-off than the people in another

country?

(a) Average income

(b) Total income

(c) GDP growth

(d) Literacy rate

A SERIES

62

43. Which one of the following statements is not correct about the Mahatma Gandhi

National Rural Employment Guarantee Act?

(a) At least one-third of the beneficiaries shall be women who have registered and

requested work under this scheme

(b) It benefits only the adult members of rural households who are willing to do public

work-related unskilled manual work at the statutory minimum wage

(c) The Union Ministry of Rural Development is monitoring the entire implementation of

this scheme in association with state governments

(d) By recent amendment to this act, it provides a legal guarantee of employment

in every financial year for around 250 days

EXPLANATION:

At least one-third beneficiaries shall be women who have registered and requested work

under the scheme. So, option (a) is correct.

It benefits only the adult members of rural households willing to do public work-related

unskilled manual work at the statutory minimum wage. So, option (b) is correct.

The Ministry of Rural Development (MRD), Government of India is monitoring the entire

implementation of this scheme in association with state governments. So, option (c) is

correct.

It provides a legal guarantee of employment in every financial year for around 100 days.

So, option (d) is not correct.

ADDITIONAL INFORMATION:

Mahatma Gandhi Employment Guarantee Act 2005 (or, NREGA No 42, later

renamed as the "Mahatma Gandhi National Rural Employment Guarantee Act",

MGNREGA), is an Indian labour law and social security measure that aims to

guarantee the 'right to work'. This act was passed in September 2005.

It aims to enhance livelihood security in rural areas by providing at least 100 days of

wage employment in a financial year to every household whose adult members

volunteer to do unskilled manual work.

The MGNREGA was initiated with the objective of "enhancing livelihood security in

rural areas by providing at least 100 days of guaranteed wage employment in a

financial year, to every household whose adult members volunteer to do unskilled

manual work".

A SERIES

63

Another aim of MGNREGA is to create durable assets (such as roads, canals, ponds

and wells).

Employment is to be provided within 5 km of an applicant's residence, and

minimum wages are to be paid.

If work is not provided within 15 days of applying, applicants are entitled to an

unemployment allowance. That is, if the government fails to provide employment, it

has to provide certain unemployment allowances to those people. Thus, employment

under MGNREGA is a legal entitlement.

MGNREGA is to be implemented mainly by gram panchayats (GPs). The involvement

of contractors is banned.

Apart from providing economic security and creating rural assets, NREGA can help

in protecting the environment, empowering rural women, reducing rural-urban

migration and fostering social equity, among others."

The law provides many safeguards to promote its effective management and

implementation. The act explicitly mentions the principles and agencies for

implementation, list of allowed works, financing pattern, monitoring and evaluation,

and most importantly the detailed measures to ensure transparency and

accountability.

SOURCE:

https://vikaspedia.in/social-welfare/rural-poverty-alleviation-1/schemes/faqs-on-

mgnrega/mahatma-gandhi-national-rural-employment-guarantee-act

44. In India, which of the following farming methods are recognised as “Global

Agricultural Heritage Systems” (GIAHS) by the Food and Agriculture Organization

(FAO)?

1. Kuttanad Below Sea Level Farming System of Kerala

2. Bamboo Drip Irrigation method in Meghalaya

3. Pampore Saffron Heritage of Kashmir

4. Koraput Traditional Agriculture of Odisha

Select the correct answer using the code given below.

(a) 1, 2 and 3 only

(b) 1, 2 and 4 only

(c) 1, 3 and 4 only

(d) 2, 3 and 4 only

A SERIES

64

EXPLANATION:

Three recognised GIAHS sites in India are

Kuttanad Below Sea Level Farming System of Kerala

Koraput Traditional Agriculture of Odisha

Pampore Saffron Heritage of Kashmir

So, option (c) is correct.

ADDITIONAL INFORMATION:

Globally Important Agricultural Heritage Systems (GIAHS) is to promote public

understanding, awareness about sustainable agriculture and to safeguard the

social, cultural, economic and environmental goods and services these provide to

family farmers, smallholders, indigenous peoples and local communities.

Hence, FAO started to identify the economic viability of the system, the identification

of environmentally sustainable strategies in the face of growing climate change, and

the empowerment of small holder/traditional family farming and indigenous

communities under Globally Important Agricultural Heritage Systems (GIAHS).

Objectives of Globally Important Agricultural Heritage Systems (GIAHS)

To identify and provide institutional support to nature friendly agricultural practices

of local and tribal populations around the world.

Capacity building of local farming communities to conserve and manage revenue

based farming in a sustainable fashion.

To mitigate risks of erosion of biodiversity and traditional knowledge, land

degradation

To strengthen conservation and sustainable use of biodiversity and natural

resources.

To reduce vulnerability to climate change and enhance sustainable agriculture and

rural development.

To achieve food security and poverty alleviation.

To promote enabling regulatory policies and incentive environments to support the

conservation, evolutionary adaptation and viability of GIAHS.

Providing incentives for local population by measures like eco-labelling, eco-tourism.

A SERIES

65

45. Consider the following statements about the “Genetic Engineering Appraisal

Committee” (GEAC):

1. It is a statutory body established under the “Biological Diversity Act, 2002”.

2. The approval of the GEAC is mandatory for the public and private research

organisations to carry out research on genetically modified organisms in India.

Which of the statements given above is/are correct?

(a) 1 only

(b) 2 only

(c) Both 1 and 2

(d) Neither 1 nor 2

EXPLANATION:

The Genetic Engineering Appraisal Committee (GEAC) is a statutory body constituted

under the ‘Rules for the Manufacture, Use /Import /Export and Storage of Hazardous

Microorganisms/Genetically Engineering Organisms or Cells, 1989’ notified under the

Environment (Protection) Act, 1986.So, statement 1 is not correct.

The approval of the GEAC is mandatory before genetically modified organisms and

products can be used commercially. So, statement 2 is not correct.

ADDITIONAL INFORMATION:

The functions of GEAC as prescribed in the Rules 1989 are as follows:

To appraise activities involving large scale use of hazardous microorganisms and

recombinants in research and industrial production from the environmental angle.

To appraise proposals relating to release of genetically engineered organisms and

products into the environment including experimental field trials.

The committee or any persons authorized by it has powers to take punitive action

under the Environment Protection Act.

The composition of the GEAC has been prescribed in Rules 1989 as given below:

Chairman- Special Secretary/Additional Secretary, Ministry of Environment, Forest

and Climate Change (MoEF&CC); Co-Chairman - Representative of Department of

Biotechnology.

Members: Representative of concerned Agencies and Departments, namely, Ministry

of Industrial Development, Department of Biotechnology and the Department of

Atomic Energy.

A SERIES

66

Expert members: Director General of Indian Council of Agricultural Research,

Director General of Indian Council of Medical Research, Director General of Council

of Scientific and Industrial Research, Director General of Health Services, Plant

Protection Adviser, Directorate of Plant Protection, Quarantine and storage,

Chairman, Central Pollution Control Board and three outside experts in individual

capacity.

Member Secretary: An official of the Ministry of Environment, Forest and Climate

Change (MoEF&CC).

SOURCE:

https://geacindia.gov.in/about-geac-india.aspx

46. Consider the following statements about the Micro, Small and Medium Enterprises

(MSMEs) in India:

1. MSMEs accounted for more than 25% of GDP and more than 40% of the overall

exports from India.

2. Public Sector Units have been mandated to procure 25% of their purchases

from MSMEs.

3. MSMEs in manufacturing sector are exempted from major labour laws in India.

Which of the statements given above is/are correct?

(a) 1 and 2 only

(b) 2 and 3 only

(c) 1 and 3 only

(d) 1, 2 and 3

EXPLANATION:

MSMEs accounted for 45 per cent of the country’s total manufacturing output, 40 per

cent of exports — and almost 30 per cent of the national GDP.So, statement 1 is

correct.

The government had directed all departments and public sector companies to purchase

at least 25% of their goods and services from such enterprises. So, statement 2 is

correct.

MSMEs in manufacturing sector are not exempted from major labour laws in India. So,

statement 3 is not correct.

A SERIES

67

SOURCE:

https://indianexpress.com/article/business/economy/india-coronavirus-lockdown-

economy-msme-6391186/

https://economictimes.indiatimes.com/news/economy/policy/government-seeks-

data-from-cpses-on-procurement-from-msme-

sector/articleshow/66662601.cms?from=mdr

47. Consider the following statements about the “National Emergency” imposed under

the Article 352 of the Indian Constitution:

1. The Parliament of India can suspend the state legislatures and can make laws

on the State subjects.

2. The proclamation of Emergency must be approved by both the Houses of

Parliament within two months from the date of its issue.

3. The Centre can alter distribution of revenue between the Union and the State

governments.

Which of the statements given above is/are correct?

(a) 1 only

(b) 2 and 3 only

(c) 1 and 3 only

(d) 2 only

EXPLANATION:

State Govt. is not suspended. Union Govt. can issue orders to the State Govt. on subjects

on the State List (something that it can’t normally do). So statements 1 is not correct.

The proclamation of Emergency must be approved by both the Houses of Parliament

within two months from the date of its issue. So statements 2 is correct.

The Centre can alter distribution of revenue between the union and the state. The Centre

can alter distribution of revenue between the union and the state. So statement 3 is

correct

ADDITIONAL INFORMATION:

When can it be imposed?

3 conditions – War, External Aggression, Armed Rebellion

Armed Rebellion was changed from Internal Disturbance on the recommendation of the

Shah Commission. Internal Disturbance was a vague term prone to misuse.

A SERIES

68

Features

Can only be proclaimed on the written advice of the Union Cabinet (not the PM).

Perhaps, this is the only place where Union Cabinet has a role.

Expires in 1 month from its issue unless approved by a Special Majority (of the

second kind*) – Not less than 2/3rd of the Members present and voting + Absolute

Majority – in both the houses of the Parliament.

If the LS is dissolved, then RS shall approve it within 1 month and the re-

constituted LS shall ratify within 30 days.

Once approved, the proclamation is extended for 6 months, which can be extended

again for 6 more months.

Not less than 1/10th of the Members of the LS(this can only be initiated in the

LS) may give notice in writing to the Speaker or President (when LS is not in

session). If there is no session, a special sitting of the LS shall be held within 14

days. If the resolution, the President has to revoke the Emergency.

Effects

Executive : State Govt. is not suspended. Union Govt. can issue orders to the State Govt. on subjects on the State List (something that it can’t normally do).

Legislature : State Legislature is not suspended. However, Parliament can make laws

on the State subjects. Such laws remain valid for 6 months after the Emergency

ceases to be.

Financial : Distribution as per the President’s will subject to approval by the

Parliament.

Effect on FR (2 clauses here)

Art. 19 automatically suspended (only in case of War and External Aggression)

President by a further order can specify other FRs that wont be operative, excepting

Art. 20 and 21.

SOURCE:

Indian Polity - Laxmikanth

48. Recently, Indian Air Force had officially inducted the indigenously developed

“Tejas” aircraft into service. Which among the following statement is not correct

about “Tejas”?

(a) It’s the lightest and smallest multirole supersonic fighter aircraft in its class

(b) It’s being the second fighter jet that got developed under LCA programme

A SERIES

69

(c) It’s capable to fire short range air-air beyond visual range missile

(d) It’s naval version LCA-MK1 Tejas is capable to perform landing over INS Shivaji

EXPLANATION:

It is the smallest lightweight, multi-role, single-engine tactical fighter aircraft in the

world.So, option (a) is correct.

It came from the Light Combat Aircraft (LCA) programme, which began in the 1980s to

replace India's ageing MiG-21 fighters. The Tejas is the second supersonic fighter

developed by Hindustan Aeronautics Limited (HAL) after the HAL HF-24 Marut. So,

option (b) is correct.

Tejas successfully demonstrated an Air-to-Air Beyond Visual Range (BVR) missile firing

capability by releasing Derby Air-to-Air BVR missile in RADAR guided mode. So, option

(c) is correct.

The Naval LCA Tejas successfully carried out its first arrested landing on the aircraft-

carrier INS Vikramaditya. So, option (d) is not correct.

SOURCE:

https://economictimes.indiatimes.com/news/defence/tejas-successfully-performs-

critical-test-for-naval-

deployment/articleshow/71115286.cms#:~:text=NEW%20DELHI%3A%20The%20n

aval%20version,navy%20for%20carrier%2Dborne%20operations.

49. The painting of “Bharat Mata” which was first seen during the Swadeshi

movement was done by

(a) Rabindranath Tagore

(b) Abanindranath Tagore

(c) Bankim Chandra Chatterjee

(d) Aurobindo Ghose

EXPLANATION:

Bharat Mata is a work painted by the Indian painter Abanindranath Tagore in 1905. The

work depicts Bharat Mata, or Mother India, in the style of a Hindu Goddess. The painting

was the first illustrated depiction of the concept, and was painted during with Swadesh

A SERIES

70

ideals during the larger Indian Independence movement.So, option (b) is correct.

ADDITIONAL INFORMATION:

Bharat Mata is depicted as a saffron-clad woman, holding a book, sheaves of paddy,

a piece of white cloth and a rosary in her four hands. The painting holds historical

significance as it is one of the earliest visualizations of Bharat Mata, or "Mother

India."

The work was painted during the Swadeshi movement. The movement began as a

response to the Partition of Bengal (1905), when Lord Curzon split the largely

Muslim eastern areas of Bengal from the largely Hindu western areas. In response,

Indian nationalists participating in the swadeshi movement resisted the British by

boycotting British goods and institutions, holding meetings and processions,

forming committees, and applying diplomatic pressure.

The painting's central figure holds multiple items associated with Indian culture and

the economy of India in the early twentieth century, such as a book, sheaves of

paddy, a piece of white cloth and a garland. Moreover, the painting's central figure

has four hands, evocative of Hindu imagery, which equates multiple hands with

immense power.

The painting has been characterized as "an attempt of humanisation of ‘Bharat

Mata’ where the mother is seeking liberation through her sons," by Jayanta

Sengupta, curator of the Indian Museum in Kolkata, India.

50. Consider the following parameters:

1. Eccentricity of the Earth’s orbit.

2. Obliquity of the Earth’s rotational axis.

3. Precession of the Earth.

Which of the parameter(s) mentioned above affects the total radiation received by

the Earth during a particular year?

(a) 1 only

(b) 1 and 2 only

(c) 2 and 3 only

(d) 1, 2 and 3

A SERIES

71

EXPLANATION:

Eccentricity is defined as the difference in shape between an ellipse and a perfect circle.

In a similar fashion to Earth's obliquity, the more uniform Earth's orbit is (more like a

perfect circle), the less difference there is in climate change throughout the year. Unlike

obliquity, eccentricity affects the entire planet approximately the same, instead of

primarily changing polar climate. The base idea with eccentricity is this: “How far away is

the earth as a whole from the sun?” If there is no eccentricity to Earth's orbit, then Earth

will remain at the same distance from the sun throughout the year, therefore producing

no climate change, seeing as how the Earth's orbit would be perfectly circular around the

sun.

So when earth is at perihelion position it receives more radiation. So statement 1 is

correct.

The precession of solstices and equinoxes is the third factor that plays a role in Earth

climate change that deals directly with the earth itself. It does not affect the radiation

received. So statement 3 is not correct.

SOURCE:

G C leong

51. The term “Schistocerca gregaria”, sometimes seen in the news recently, is a

(a) world’s largest cave fish discovered in the north eastern part of India

(b) dangerous trans-boundary locust that attacks on the standing crops

(c) new species of stegosaurus dated to 168 million years ago

(d) newly discovered bacterium that had naturally evolved to eat plastic

52. Consider the following pairs:

Biodiversity heritage State

site

1. Ambaraguda - Andhra Pradesh

2. Ameenpur lake - Telangana

3. Hogrekan - Karnataka

Which of the pairs given above is/are correctly matched?

(a) 1 only

(b) 1 and 2 only

(c) 2 and 3 only

(d) 1, 2 and 3

A SERIES

72

EXPLANATION:

Ambaraguda islocated in Shimoga, Karnataka. This portion of Western Ghats has

precious reserves of unique and endemic plant species which helped earn the status of a

Biodiversity Heritage Site. Ambaraguda is located between Sharavathi Wildlife Sanctuary

and Someshwara Wildlife Sanctuary.So, pair 1 is not correct.

Ameenpur Lake islocated in Sangareddy, Telangana. First water body to be recognised as

a Biodiversity Heritage Site, Ameenpur Lake is a man-made lake dating more than 300

years old. The lake is home to many resident and migratory birds, such as flamingos,

egrets, herons, cormorants, kingfishers, and river terns, to name a few. And because of

high number of migratory species, Ameenpur Lake earned the status of a BHS. The site is

every birdwatcher's delight.So, pair 2 is correct.

Hogrekan islocated in Chikmagalur, KarnatakaThe Shola vegetation of Hogrekan is home

to a number of unique medicinal plants. Shola forests are stunted tropical montane

forest, insterspersed by undulating grasslands. Due to its proximity to Bhadra Wildlife

Sanctuary and Yemmedode Tiger Reserve, it also serves as an important wildlife corridor.

A lot of floral species from this site have medicinal values and are endemic in nature.

According to some studies, these Shola grasslands are thousands of years old.So, pair 3

is correct.

53. With reference to the Economic Survey 2019-20, consider the following

statements:

1. The public expenditure as percentage of GDP for education increased much

faster than that on health during 2014-15 to 2019-20.

2. As a proportion of GDP, the expenditure on social services decreased during

2014-15 to 2019-20.

Which of the statements given above is/are correct?

(a) 1 only

(b) 2 only

(c) Both 1 and 2

(d) Neither 1 nor 2

EXPLANATION:

The expenditure on education, as percentage of GDP increased from 2.8% to 3.1%

between 2014-15 and 2019-20 (BE), expenditure on health increased from 1.2% to 1.6%

during the same period as percentage of GDP. Hence, The public expenditure as

percentage of GDP for education increased not much faster than that on health during

2014-15 to 2019-20. So, statement 1 is not correct.

A SERIES

73

According to the Economic Survey, as a proportion of gross domestic product (GDP), the

expenditure on social services has registered an increase of 1.5 percentage points during

the period 2014-15 to 2019-20 from 6.2% to 7.7%. So, statement 2 is not correct.

SOURCE:

https://economictimes.indiatimes.com/news/economy/indicators/govt-spend-on-

social-services-up-by-1-5-in-six-years-eco-

survey/articleshow/73798267.cms#:~:text=While%20the%20expenditure%20on%20e

ducation,the%20pre%2DBudget%20document%20notes.

54. With reference to the British rule in India, which one of the following peasant

movements is related to the “Nij system” of cultivation?

(a) Rangpur uprising

(b) Deccan Uprising

(c) Indigo Revolt

(d) Moplah Rebellion

EXPLANATION:

In Nij Cultivation system, the planter produced indigo on lands which were under his

direct control. The planter either bought the land or rented it from other zamindars. He

directly employed labourers to produce indigo.

In Bengal, the indigo planters, nearly all Europeans, exploitedthe local peasants by

forcing them to grow indigo on theirlands instead of the more paying crops like rice. The

plantersforced the peasants to take advance sums and enter intofraudulent contracts

which were then used against the peasants.The planters intimidated the peasants

through kidnappings,illegal confinements, flogging, attacks on women and

children,seizure of cattle, burning and demolition of houses anddestruction of crops. It

resulted in the form of Indigo revolt by peasants. So, option (c) is correct.

ADDITIONAL INFORMATION:

Rangpur Dhing – The Uprising

Debi Singh practised extremely harsh measures against the peasants.When the

peasants sent a petition to the company asking for relief, it did not pay any heed to

the farmers’ grievances. This led to the farmers taking things into their own hands.

A SERIES

74

The uprising started on January 18, 1783, when peasants and zamindars took

control of the Parganas of Kakina, Kazirhat and Tepa in district Rangpur.

They attacked the courts, looted grains and released prisoners. For a whole 5 weeks,

these areas were under the control of the rebels who appointed a nawab and other

officials for running a parallel government.

One of the main leaders of the revolt was Kena Sarkar. They forbid all revenue

payments to the company.The uprising spread to Dinajpur also.

Ultimately, the uprising was put down by the British and many rebels were killed.

Moplah Uprisings

Hike in revenue demand and reduction of field size, coupled with the oppression of

officials, resulted in widespread peasant unrest among the Moplahs of Malabar.

Twenty-two rebellions took place between 1836 and 1854. None, however, proved

successful.

The second Moplah uprising occurred after the Moplah came to be organised by the

Congress and the Khilafat supporters during the Non-cooperation Movement. But

Hindu- Muslim differences distanced the Congress and the Moplahsfrom each other.

By 1921, the Moplahs had been subdued.

Deccan Riots

The ryots of Deccan region of western India suffered heavy taxation under the

Ryotwari system. Here again the peasants found themselves trapped in a vicious

network with the moneylender as the exploiter and the main beneficiary.

Thesemoneylenders were mostly outsiders—Marwaris or Gujaratis.

The conditions had worsened due to a crash in cotton prices after the end of the

American Civil War in 1864, the Government’s decision to raise the land revenue by

50% in1867, and a succession of bad harvests.

In 1874, the growing tension between the moneylenders and the peasants resulted

in a social boycott movementorganised by the ryots against the “outsider”

moneylenders.

The ryots refused to buy from their shops. No peasant would cultivate their fields.

The barbers, washermen, shoemakers would not serve them. This social boycott

spread rapidly to the villages of Poona, Ahmednagar, Sholapur and Satara. Soon the

social boycott was transformed into agrarian riots withsystematic attacks on the

moneylenders’ houses and shops.

The debt bonds and deeds were seized and publicly burnt. The Government

succeeded in repressing the movement. As a conciliatory measure, the Deccan

A SERIES

75

Agriculturists Relief Act was passed in 1879. This time also, the modern nationalist

intelligentsia of Maharashtra supported the peasants’ cause.

SOURCE:

A brief history of modern India by Spectrum

55. Consider the following statements about the “State Disaster Management

Authority” (SDMA) in India:

1. All State Governments in India are mandated under the Disaster Management

Act, 2005 to establish a State Disaster Management Authority.

2. The SDMA of a particular Indian state is headed by the Chief Minister of that

state.

3. In case of Union Territory, the Disaster Management Authority will be headed

by the President of India.

4. The Chairperson of District Disaster Management Authority (DDMA) will be the

Collector or District Magistrate or Deputy Commissioner of that district.

Which of the statements given above are correct?

(a) 1, 2 and 3 only

(b) 1, 2 and 4 only

(c) 1, 3 and 4 only

(d) 2, 3 and 4 only

EXPLANATION:

All State Governments are mandated under Section 14 of the Disaster Management Act,

2005 to establish a State Disaster Management Authority (SDMA). So, statement 1 is

correct.

The SDMA consists of the Chief Minister of the State, who is the Chairperson, and no

more than eight members appointed by the Chief Minister. So, statement 2 is correct.

In case of Union Territory, the Disaster Management Authority will be headed not by the

President of India but by Lt Governor. For pondicherry it is headed by chief minister. So,

statement 3 is not correct.

The Chairperson of District Disaster Management Authority (DDMA) will be the Collector

or District Magistrate or Deputy Commissioner of the district. The elected representative

of the area is member of the DDMA as an ex officio co-Chairperson. So, statement 4 is

correct.

A SERIES

76

ADDITIONAL INFORMATION:

The Disaster Management Act, 2005 has been enacted as the central Act to deal

with the management of disasters. This act envisaged a three tier Disaster

Management structure in India at National, States and District levels. Under the act,

the NDMA, SDMA, NEC, NDRF, NIDM and disaster related funds were established.

National Authority

The Act calls for the establishment of National Disaster Management Authority

(NDMA), with the Prime Minister of India as chairperson.

The NDMA may have no more than nine members including a Vice-Chairperson. The

tenure of the members of the NDMA shall be five years. The NDMA is responsible for

"laying down the policies, plans and guidelines for disaster management" and to

ensure "timely and effective response to disaster". Under section 6 of the Act it is

responsible for laying "down guidelines to be followed by the State Authorities in

drawing up the State Plans".

National Executive Committee

The Act under Section 8 enjoins the Central Government to Constitute a National

Executive Committee (NEC) to assist the National Authority.

The NEC is composed of Secretary level officers of the Government of India in the

Ministries of home, agriculture, atomic energy, defence, drinking water supply,

environment and forests, finance (expenditure), health, power, rural development,

science and technology, space, telecommunication, urban development, and water

resources, with the Home secretary serving as the Chairperson, ex officio.

The Chief of the Integrated Defence Staff of the Chiefs of Staff Committee, is an ex

officio member of the NEC.

The NEC under section of the Act is responsible for the preparation of the National

Disaster Management Plan for the whole country and to ensure that it is "reviewed

and updated annually".

State Disaster Management Authority

All State Governments are mandated under Section 14 of the act to establish a State

Disaster Management Authority (SDMA).

The SDMA consists of the Chief Minister of the State, who is the Chairperson, and

no more than eight members appointed by the Chief Minister.

State Executive Committee is responsible (Section 22) for drawing up the state

disaster management plan, and implementing the National Plan.

A SERIES

77

The SDMA is mandated under section 28 to ensure that all the departments of the

State prepare disaster management plans as prescribed by the National and State

Authorities.

District Disaster Management Authority

The Chairperson of District Disaster Management Authority (DDMA) will be the

Collector or District Magistrate or Deputy Commissioner of the district. The elected

representative of the area is member of the DDMA as an ex officio co-Chairperson,

(Section 25).

National Disaster Response Force (NDRF)

The Section 44–45 of the Act provides for constituting a National Disaster Response

Force "for the purpose of specialist response to a threatening disaster situation or

disaster" under a Director General to be appointed by the Central Government.

SOURCE:

https://vikaspedia.in/social-welfare/disaster-management-1/disaster-management-

in-india

56. With reference to the religious history of India, “Pubbaseliya”, “Aparaseliya” and

“Gokulika” were the sub-sects of

(a) Buddhism

(b) Jainism

(c) Vaishnavism

(d) Shaivism

EXPLANATION:

The Dīpavaṃsa ("Chronicle of the Island") is the oldest historical record of Sri Lanka. The

chronicle is believed to be compiled from Atthakatha and other sources around the 3rd to

4th century CE. Together with the Mahavamsa, it is the source of many accounts of

ancient history of Sri Lanka and India. Its importance resides not only as a source of

history and legend, but also as an important early work in Buddhist and Pali

literature.Gokulika is the sub-sect of Buddhism.The Dipavamsa lists the following six

schools without identifying the schools from which they arose:

Hemavatika (Sanskrit: Haimavata)

Rajagiriya

A SERIES

78

Siddhatthaka

Pubbaseliya

Aparaseliya (Sanskrit: Aparaśaila)

Apararajagirika

So, option (a) is correct.

57. Which one of the following climatic types has the highest vertical depth for

weathering?

(a) Tundra

(b) Equatorial

(c) Savanna

(d) Desert

EXPLANATION:

Equatorial temperature has high vertical depth of weathering because many important

factors of weathering act in high intensity in equatorial region. For example equatorial

region has high temperature, high rainfall.So option (b) is correct

ADDITIONAL INFORMATION:

Weathering

The process of wearing away of earth’s surface is commonly known as denudation &

is generally carried out in four phases Weathering, Erosion, Transportation &

Deposition

Warm wet climate promotes rapid chemical weathering while dry climate provide

good conditions for physical weathering

Chemical Weathering

Extremely slow & gradual decomposition of rocks due to exposer to air & water

For example Granite when exposed to weather is found to be rough surfaced

because it is mainly made up of Quartz, Feldspar & Mica; Feldspar is more quickly

weathered than Quartz hence is worn away, eventually leaving loosened quartz

crystals.

Regolith →Weathered material from the rock or mineral remains of decomposed

rocks.

A SERIES

79

When a soil cover on the rock exists, chemical weathering of the rock enhances

because the soil absorbs rain water & keeps the underlying rock in contact with this

moisture.

Rain water absorbs organic acids from the soil & thus become a stronger weathering

agent than pure water acting on a bare rock.

Physical Weathering

Also known as Mechanical Weathering

Disintegration by Mechanical Process

Types of physical weathering → By insolation, by Frost

Biological Weathering

By Men, Animals, Insects & Vegetation

vegetation grows into crevices of rock cracks or in courtyards or building walls

SOURCE:

G C Leong

58. Consider the following statements about the “Interstellar object”.

1. It is an astronomical object such as an asteroid, a comet or a rogue planet that

is located in the interstellar space.

2. Oumuamua is the first known interstellar object detected passing through the

Solar System.

Which of the statements given above is/are correct?

(a) 1 only

(b) 2 only

(c) Both 1 and 2

(d) Neither 1 nor 2

EXPLANATION:

An interstellar object is an astronomical object (such as an asteroid, a comet, or a rogue

planet, but not a star) that is located in interstellar space and is not gravitationally

bound to a star.So, statement 1 is correct.

Oumuamua is the first known interstellar object detected passing through the Solar

System.So, statement 2 is correct.

A SERIES

80

ADDITIONAL INFORMATION:

An interstellar object is an astronomical object (such as an asteroid, a comet, or a

rogue planet, but not a star) that is located in interstellar space and is not

gravitationally bound to a star.

This term can also be applied to an object that is on an interstellar trajectory but is

temporarily passing close to a star, such as certain asteroids and comets (including

exocomets. In the latter case, the object may be called an interstellar interloper.

The first interstellar object to be discovered in the Solar System was 1I/ʻOumuamua

in 2017. The second was 2I/Borisov in 2019.

They both have significant hyperbolic excess velocity proving they did not originate

in the Solar System.

In addition ʻOumuamua has an orbital eccentricity of about 1.2 and Borisov has an

orbital eccentricity of about 3.3, proving neither is gravitationally bound to the Sun.

SOURCE:

https://solarsystem.nasa.gov/asteroids-comets-and-meteors/comets/oumuamua/in-

depth/

59. With reference to Un-Plastic Collective (UPC), consider the following statements

1. It is a voluntary initiative launched by the United Nations Environment

Programme, WWF India and Confederation of Indian Industry.

2. It aims to address ways to mitigate plastic pollution and move towards a

circular economy through corporate commitments.

Which of the statements given above is/are correct?

(a) 1 only

(b) 2 only

(c) Both 1 and 2

(d) Neither 1 nor 2

EXPLANATION:

The Un-plastic Collective (UPC) is co-founded by the Confederation of Indian Industry

(CII), United Nations Environment Programme (UNEP) and WWF-India.So, statement 1

is correct.

The Collective seeks to minimise externalities of plastics on the ecological and social

health of our planet. The word ‘un-plastic’ specifically refers to moving all types of

plastics in a circular economy and removing unnecessary plastic in the long run, while

A SERIES

81

using sustainable alternative materials. So, statement 2 is correct.

ADDITIONAL INFORMATION:

The UPC is a voluntary initiative. As part of this companies set time-bound, public targets

to:

Eliminate unnecessary use of plastic

Reuse and circulate plastics through the economy

Replace with sustainable alternative or recycled plastics

Turn commitments to meaningful, measurable action

Implement EPR and improvements in materials management and recovery

Undertake plastic incentivisation, baseline assessment

SOURCE:

https://www.wwfindia.org/about_wwf/making_businesses_sustainable/un_plastic_coll

ective/

60. Which one of the following countries lies completely above the Tropic of Cancer?

(a) Qatar

(b) Mexico

(c) Haiti

(d) Fiji

EXPLANATION:

Moving east of the Prime Meridian, the Tropic of Cancer passes through the following

Countries:

Algeria

Niger

Libya

Egypt

Saudi Arabia

UAE (Abu Dhabi)

Oman

India

Bangladesh

A SERIES

82

Myanmar

China

Taiwan

Mexico

Bahamas

Western Sahara (area claimed by Morocco; Sahrawi Arab Democratic Republic)

Mauritania

Mali

Qatar is above tropic of cancer. So option (a) is correct.

61. Consider the following statements about the “National Pension System” (NPS) in

India:

1. It is a pension cum investment scheme launched by Government of India to

provide old age security to Citizens of India.

2. Any Indian Citizen between 18 and 60 years of age can join NPS.

3. It is compulsory for all government employees to enrol in this scheme.

4. Government of India will contribute 50% of the premium paid.

Which of the statements given above is/are correct?

(a) 1 only

(b) 1 and 2 only

(c) 2, 3 and 4 only

(d) 1, 2, 3 and 4

EXPLANATION:

National Pension System (NPS) is a pension cum investment scheme launched by

Government of India to provide old age security to Citizens of India. So, statement 1 is

correct.

Any Indian citizen between 18 and 60 years can join NPS.So, statement 2 is correct.

It is not compulsory for all government employees to enrol in this scheme.The Central

Government had introduced the National Pension System (NPS) with effect from January

1, 2004 (except for armed forces). All the employees of Central Autonomous Bodies who

have joined on or after the above mentioned date are also mandatorily covered under

Government sector of NPS.Central Government/CABs employee contributes towards

pension from monthly salary along with matching contribution from the employer.

Subsequent to Central Government, various State Governments adopted this architecture

and implemented NPS with effect from different dates. A State Autonomous Body (SAB)

can also adopt NPS if the concerned State Government/UT have adopted the NPS

A SERIES

83

architecture and initiated implementation of the same. State Government/SABs

employees also contribute towards pension from monthly salary along with matching

contribution from the employer.So, statement 3 is not correct.

The government will not contribute to the NPS account.So, statement 4 is not correct.

ADDITIONAL INFORMATION:

NPS is a government-sponsored pension scheme. It was launched in January 2004

for government employees.

It was extended to all citizens of Indian on voluntary basis from May 2009 and to

corporates in December 2011 and to Non-Resident Indians in October 2015.

PFRDA is the statutory Authority established by an enactment of the Parliament, to

regulate, promote and ensure orderly growth of the NPS and pension schemes to

which this Act applies.

The scheme allows subscribers to contribute regularly in a pension account during

their working life.

On retirement, subscribers can withdraw a part of the corpus in a lumpsum and use

the remaining corpus to buy an annuity to secure a regular income after retirement.

Any Indian citizen between 18 and 60 years can join NPS.

The only condition is that the person must comply with know your customer (KYC)

norms.

An NRI can join NPS. However, the account will be closed if there is a change in the

citizenship status of the NRI.

Now, any Indian citizen, resident or non-resident and OCIs are eligible to join NPS

till the age of 65 years.

SOURCE:

https://npscra.nsdl.co.in/all-faq-about-

nps.php#:~:text=National%20Pension%20System%20(NPS)%20is,and%20regulated%2

0market%2Dbased%20return.

62. Under the National Policy on Biofuels of 2018, which of the following crops will be

used as raw material for the production of Ethanol?

1. Sugarcane juice and rotten Potatoes

2. Sugar containing materials like Sugar Beet and sweet Sorghum

3. Starch containing materials like Corn and Cassava

4. Damaged food grains like Wheat and broken Rice

A SERIES

84

Select the correct answer using the code given below.

(a) 1 and 2 only

(b) 3 and 4 only

(c) 1, 2 and 3 only

(d) 1, 2, 3 and 4

EXPLANATION:

The National Policy on Biofuels of 2018 expands the scope of raw material for ethanol

production by allowing use of Sugarcane Juice, Sugar containing materials like Sugar

Beet, Sweet Sorghum, Starch containing materials like Corn, Cassava, Damaged food

grains like wheat, broken rice, Rotten Potatoes, unfit for human consumption for ethanol

production. So, option (d) is correct.

ADDITIONAL INFORMATION:

Salient Features of National Policy on Biofuels of 2018:

The Policy categorises biofuels as "Basic Biofuels" viz. First Generation (1G)

bioethanol & biodiesel and "Advanced Biofuels" - Second Generation (2G) ethanol,

Municipal Solid Waste (MSW) to drop-in fuels, Third Generation (3G) biofuels, bio-

CNG etc. to enable extension of appropriate financial and fiscal incentives under

each category.

The Policy expands the scope of raw material for ethanol production by allowing use

of Sugarcane Juice, Sugar containing materials like Sugar Beet, Sweet Sorghum,

Starch containing materials like Corn, Cassava, Damaged food grains like wheat,

broken rice, Rotten Potatoes, unfit for human consumption for ethanol production.

Farmers are at a risk of not getting appropriate price for their produce during the

surplus production phase. Taking this into account, the Policy allows use of surplus

food grains for production of ethanol for blending with petrol with the approval of

National Biofuel Coordination Committee.

With a thrust on Advanced Biofuels, the Policy indicates a viability gap funding

scheme for 2G ethanol Bio refineries of Rs.5000 crore in 6 years in addition to

additional tax incentives, higher purchase price as compared to 1G biofuels.

The Policy encourages setting up of supply chain mechanisms for biodiesel

production from non-edible oilseeds, Used Cooking Oil, short gestation crops.

Roles and responsibilities of all the concerned Ministries/Departments with respect

to biofuels has been captured in the Policy document to synergise efforts.

A SERIES

85

SOURCE:

https://pib.gov.in/Pressreleaseshare.aspx?PRID=1532265

63. Consider the following statements about the “Right to Information Act, 2005”:

1. Political parties will not come under the ambit of the Right to Information Act.

2. Citizens can access to court records under provision of this Act.

Which of the statements given above is/are correct?

(a) 1 only

(b) 2 only

(c) Both 1 and 2

(d) Neither 1 nor 2

EXPLANATION:

In a judgement Supreme cout has said that political parties come under RTI. So

statement 1 is not cocrrect

Citizens cannot access to court records under provision of the Act. Only the office og CJI

comes under RTI. So statement 2 is not correct.

ADDITIONAL INFORMATION:

Objectives of the Act

To empower the citizens

To promote transparency and accountability

To contain corruption and

To enhance people’s participation in democratic process.

Reasons for Adoption of Information Act

The factors responsible for adoption of information act are as follows-

Corruption and scandals

International pressure and activism

Modernization and the information society

Features of the Act

Section 1(2) : It extends to the whole of India except the State of Jammu and

Kashmir.

Section- 2 (f): "Information" means any material in any form, including Records,

Documents, Memos, e-mails, Opinions, Advices, Press releases, Circulars, Orders,

A SERIES

86

Logbooks, Contracts, Reports, Papers, Samples, Models, Data material held in any

electronic form and information relating to any private body which can be accessed

by a Public Authority under any other law for the time being in force.

Section- 2(j) : "Right to Information" means the right to information accessible under

this Act which is held by or under the control of any public authority and includes

the right to:

o Inspection of work, documents, records;

o Taking notes, extracts or certified copies of documents or records;

o Taking certified samples of material;

o Obtaining information in the form of diskettes, floppies, tapes, video

cassettes or in any other electronic mode or through printouts where such

information is stored in a computer or in any other device.

Section 4 of the RTI Act requires suomotu disclosure of information by each public

authority. However, such disclosures have remained less than satisfactory.

Section 8 (1) mentions exemptions against furnishing information under RTI Act.

Section 8 (2) provides for disclosure of information exempted under Official Secrets

Act, 1923 if larger public interest is served.

The Act also provides for appointment of Information Commissioners at Central and

State level. Public authorities have designated some of its officers as Public

Information Officer. They are responsible to give information to a person who seeks

information under the RTI Act.

Time period: In normal course, information to an applicant is to be supplied within

30 days from the receipt of application by the public authority.

If information sought concerns the life or liberty of a person, it shall be

supplied within 48 hours.

In case the application is sent through the Assistant Public Information Officer or it

is sent to a wrong public authority, five days shall be added to the period of thirty

days or 48 hours, as the case may be

SOURCE:

https://www.thehindu.com/news/national/political-parties-yet-to-comply-with-rti-

act/article26601174.ece

A SERIES

87

64. With reference to the views of Indian Freedom fighters on education system,

consider the following statements:

1. Mahatma Gandhi was highly critical of Western education while Rabindranath

Tagore wanted to combine elements of modern Western civilisation with what

he saw as the best within Indian tradition.

2. ‘Nai Talim’ is a new education system proposed by Mahatma Gandhi.

3. Rabindranath Tagore started a school ‘Santiniketan’ where he emphasized the

need to teach Science and Technology, Art, Music and Dance.

Which of the statements given above are correct?

(a) 1 and 2 only

(b) 2 and 3 only

(c) 1 and 3 only

(d) 1, 2 and 3

EXPLANATION:

M.K Gandhi was highly critical of Western civilisation and its worship of machines and

technology. In Gandhiji’s own words Hind Swaraj ‘is a severe condemnation of modern

civilization’. Gandhiji’s negative opinion about modern civilization extended to all its

aspects. Modern technology, machine being the fundamental constituting unit of it; and

Western social institutions, particularly those pertaining to law and medicine, came in for

the most severe criticism.So, statement 1 is correct.

NaiTalim is a principle which states that knowledge and work are not separate. Mahatma

Gandhi promoted an educational curriculum with the same name based on this

pedagogical principle.Mahatma Gandhi has given his scheme of NaiTalim (New

Education) in a well formulated approach to education in 1937 in his news paper

'Harijan'.So, statement 2 is correct.

Rabindranth Tagore started ‘Santiniketan’ where he emphasised the need to teach

science and technology, art, music, dance. So, statement 3 is correct.

65. Consider the following pairs:

Festival State

1. Thalfavang Kut - Odisha

2. Ashadhi Beej - Assam

3. Lai Haraoba - Manipur

A SERIES

88

Which of the pairs given above is/are correctly matched?

(a) 3 only

(b) 2 and 3 only

(c) 1 and 2 only

(d) 1, 2 and 3

EXPLANATION:

Thalfavang Kut is a Mizoram’s Harvest Festival. It is celebrated in the months of

November this is also a festival concerning harvest.This marks the onset of harvest.

Various communities and tribes gather around and pitch in their collections in the form

of a feast as they merrily spend the time. Cultural activities are highlighted in the form of

traditional games and sing alongs or dance and music.So, pair 1 is not correct.

Ashadi Beej or Ashadhi Bij is observed on the second day of the Shukla Paksha or waxing

phase of moon in Ashada month. It is the Kutchi New Year. This Hindu New Year is

observed in the Kutch region in Gujarat. Ashadi Beej is mostly a traditional affair and is

a celebration of rains. During Ashadhi-beej, farmers check the moisture in the

atmosphere to help predict which crop would do best in coming monsoon. So, pair 2 is

not correct.

Lai Haraoba is one of the important festivals of Manipur, and it is celebrated to pay

respect and honour to the local traditional deities and ancestors. The festival is usually

celebrated by the Meitei community and is also referred to as the ‘Festivity of the

Gods’.So, pair 3 is correct.

SOURCE:

http://newsonair.com/Main-News-Details.aspx?id=377056

https://pib.gov.in/Pressreleaseshare.aspx?PRID=1577160

66. Consider the following statements about the Household Air Pollution in India:

1. It is the emissions of PM 10 generated by the burning of solid fuels in

households.

2. By recent studies, it is the single largest cause of Ambient Air Pollution.

3. This pollution predominantly prevails in the rural part of India.

Which of the statements given above is/are correct?

(a) 2 only

(b) 1 and 2 only

A SERIES

89

(c) 1 and 3 only

(d) 1, 2 and 3

EXPLANATION:

The emissions of PM 2.5 generated by the burning of solid fuels in households is termed

Household Air Pollution (HAP).So, statement 1 is not correct.

Household air pollution is the single largest cause of Ambient Air Pollution. So,

statement 2 is correct.

This pollution prevails both in the rural and urban part of India. So, statement 3 is not

correct.

ADDITIONAL INFORMATION:

Household Air Pollution and how dangerous is it?

Fine particulate matter refers to particles or droplets with a diameter of 2.5

micrometers (0.000001 metres) or less, and is also known as PM2.5.

The emissions of PM 2.5 generated by the burning of solid fuels in households is

termed Household Air Pollution (HAP).

The study claims that approximately 800,000 premature deaths occur in India every

year as a result of exposure to HAP indoors.

Moreover, the HAP produced indoors travels outdoors, and becomes a contributor to

ambient air pollution, with around 300,000 more premature deaths per year

attributable to exposure to outdoor HAP.

The full impact of HAP is thus composed of the exposures to HAP 1) inside and

around a given house and 2) from the household contribution to ambient air

pollution states the study.

The median estimate for the contribution of HAP is, according to the study, around

30%, far greater than that of industries (2%-10%), power plants (8%-15%), and

transportation (8%-11%).

The contribution of HAP to premature mortality is, as per the median across all

studies, 58% higher than premature mortality due to coal use, 303% higher than

that due to open burning, and 1,056% higher than that due to transportation.

SOURCE:

https://indianexpress.com/article/explained/ujjwala-yojana-india-clean-fuel-air-

pollution-5917902/

A SERIES

90

67. Why is the Mediterranean region prone to more local winds?

(a) It is located in the shifting of pressure belts

(b) The region experiences winter rainfall

(c) Its topography is very complex

(d) Only recently, the region became a sea

EXPLANATION:

The Mediterranean climatic region in Europe experiences many local winds due to the

topography of the region with the Alps in the North, the Sahara desert in the South, the

continental interiors in the East and the open Atlantic in the west. So option (c) is

correct.

ADDITIONAL INFORMATION:

Sirocco

This is a hot, dry and dusty wind.

It originates in the Sahara desert and might occur at any time of the year but is

mostly witnessed during spring.

Normally it lasts only for a few days.

It blows from the desert interiors of Sahara into the Mediterranean Sea and is

usually associated with the depressions from the Atlantic ocean.

After crossing the Sea, it is slightly cooled by the absorption of water vapour but is

hot enough that it withers vegetation and crops of the region.

Hence it is also called "Blood Rain" because it is carrying the red dust of the Sahara

desert.

Mistral

Mistral is a cold wind from the north.

It rushes down the Rhone valley and its velocity is intensified by the funnelling effect

in the valley between the Alps and the Central Massif[Plateau in France].

In some extreme cases, the velocity of the wind is so high that trains can be derailed

and trees are uprooted.

In winter, if the Mistral is frequent the temperatures could go below the freezing

point.

Other local winds

Bora: Cold north-easterly wind along the Adriatic coast.

Tramontana and Gregale: cold winds in the Mediterranean Sea.

A SERIES

91

SOURCE:

G C Leong

68. Consider the following statements about “YadaYada” Virus which seems to be a

novel virus that got detected in Australian mosquitoes:

1. It’s a kind of small, single stranded RNA virus.

2. It is capable to infect only the mosquitoes and not the humans.

Which of the statements given above is/are correct?

(a) 1 only

(b) 2 only

(c) Both 1 and 2

(d) Neither 1 nor 2

EXPLANATION:

YadaYadavirus is an alphavirus, a group of viruses that the researchers described as

small, single-stranded positive-sense RNA viruses.So, statement 1 is correct.

They are transmitted primarily by mosquitoes and (are) pathogenic in their vertebrate

hosts. Unlike some other alphaviruses, YadaYada does not pose a threat to human

beings. So, statement 2 is correct.

SOURCE:

https://indianexpress.com/article/explained/this-word-means-yada-yada-virus-

6222274/

69. With reference to the Ancient history of India, consider the following statements

about the practise of agriculture during Mahajanapads:

1. Introduction of wooden ploughs in the agricultural field increased the

agricultural production.

2. Transplanting technique was used in the production of paddy.

3. Slaves were widely used for agricultural activities.

A SERIES

92

Which of the statements given above are correct?

(a) 1 and 2 only

(b) 2 and 3 only

(c) 1 and 3 only

(d) 1, 2 and 3

EXPLANATION:

Mahajanapadas used iron plough. So statement 1 is not correct.

Transplanting technique was used in production of paddy. So statement 2 is correct.

Slaves were used widely in agriculture. So statement 3 is correct.

These three were he important reason for agriculture development.

ADDITIONAL INFORMATION:

Around 600 B.C two major changes occurred in the agriculture. One was an increase in the

use of iron ploughshare. By using iron ploughshare heavy, clayey soil could be turned over

better than with a wooden ploughshare. This led to an increase in production of grains.

Secondly, transplantation of paddy began during this time. In this process, instead of

scattering seed on the ground, saplings were grown and then planted in the fields. This led

to increased production, as many more plants survived. As a result of these two changes,

the agriculture flourished in the Mahajanapadas.

SOURCE:

History class 6 ncert

70. Consider the following statements about the VidyaDaan scheme which was

recently launched by the Government of India:

1. It is a program of the Union Ministry of Human Resource Development.

2. It is a common national program for contribution of e-learning content for

school and higher education levels.

3. The program is aimed at integrating digital education with schooling for

augmented learning.

Which of the statements given above is/are correct?

(a) 1 only

(b) 1 and 2 only

A SERIES

93

(c) 2 and 3 only

(d) 1, 2 and 3

EXPLANATION:

VidyaDaan scheme is a program by the Union Ministry of Human Resource Development.

So, statement 1 is correct.

VidyaDaan is conceptualised as a common national program for individuals &

organisations across the country to donate/contribute e-learning resources for both

school and higher education to ensure continuity of quality learning. So, statement 2 is

correct.

The program is aimed at integrating digital education with schooling for augmented

learning. So, statement 3 is correct.

ADDITIONAL INFORMATION:

VidyaDaan 2.0

Ministry of HRD launched national program VidyaDaan 2.0 for inviting e-learning

Content contributions.

VidyaDaan is a common national programme to develop and contribute e-learning

content and a chance to be recognized nationally.

VidyaDaan is conceptualised as a common national program for individuals and

organizations across the country to contribute e-learning resources for both school

and higher education to ensure continuity of quality learning.

The content will be used on the DIKSHA app to help millions of children across the

country to continue their learning anytime and anywhere.

SOURCE:

https://timesofindia.indiatimes.com/india/mhrd-launches-vidyadaan-2-0-to-create-e-

learning-content/articleshow/75298766.cms

71. When a spacecraft travelling in a straight line towards the Core of the Sun,

arrange the following in the order as the spacecraft would witness:

1. Convection Zone

2. Chromosphere

A SERIES

94

3. Photosphere

4. Corona

Select the correct answer using the code given below.

(a) 2–4–3–1

(b) 4–3–2–1

(c) 4–2–3–1

(d) 1–2–3–4

EXPLANATION:

So option (c) is correct.

ADDITIONAL INFORMATION:

What is Corona?

The corona is the outermost part of the Sun’s atmosphere. It is the aura of plasma

that surrounds the Sun and other stars.

The Sun’s corona extends millions of kilometres into outer space and is most easily

seen during a total solar eclipse, but it is also observable with a coronagraph.

Spectroscopy measurements indicate strong ionization in the corona and a plasma

temperature in excess of 1000000 Kelvin much hotter than the surface of the Sun.

A SERIES

95

Predicting in advance

The Predictive Solar Surface Flux Transport model developed by the CESSI team can

predict the shape of the corona well in advance.

The researchers can predict the large-scale structure of the Sun’s corona up to two

months in advance.

This model gives advance knowledge and a large window of preparedness for space

weather driven by coronal magnetic fields.

SOURCE:

https://www.thehindu.com/sci-tech/science/scientists-unveil-most-detailed-images-

of-suns-surface-showing-boiling-plasma/article30692287.ece

72. Which of the following financial entities are regulated by the Securities and

Exchange Board of India?

1. Mutual funds

2. Alternative investment bonds

3. Real estate investment trusts

4. Investment banks

Select the correct answer using the code given below.

(a) 1 and 4 only

(b) 2 and 3 only

(c) 1, 2 and 3 only

(d) 1, 2, 3 and 4

EXPLANATION:

Alternative Investments Funds come under the SEBI (Alternative Investment Fund)

Regulations 2012. Under this regulatory regime, the funds which come under AIF are as

follows:

Venture Capital Funds

PIPE (Private Investment in Public Equity ) Funds

Private Equity Fund

Debt Funds

Infrastructure Equity Fund

Real Estate Fund

SME Fund

Social Venture Funds

A SERIES

96

Strategy Fund (Residual Category, including all varieties of funds such as hedge

funds, if any).

SEBI perform the function of registration and regulation of the working of venture capital

funds and collective investment schemes including mutual funds. It also regulates

investment banks. So, option (d) is correct.

73. “Financial Secrecy Index”, sometimes seen in the news recently, is released by

(a) World Bank

(b) World Economic Forum

(c) Tax Justice Network

(d) Economist Intelligence Unit

EXPLANATION:

The Financial Secrecy Index ranks jurisdictions according to their secrecy and the scale

of their offshore financial activities. A politically neutral ranking, it is a tool for

understanding global financial secrecy, tax havens or secrecy jurisdictions, and illicit

financial flows or capital flight.The index was unveiled by the Tax Justice Network (TJN).

The index is based on twenty secrecy indicators which can be grouped into four broad

dimensions.So, option (c) is correct.

ADDITIONAL INFORMATION:

Switzerland no longer retains the top position in the newly released financial secrecy

index (FSI-2020) – a slot occupied by it since 2011. Cayman Island ranked first,

moving up two slots from the 2018 ranking. US continued to retain its second

position.

Tax Justice Network (TJN), an independent international network which pushes for

transparency, ranks countries on the size and secretiveness of their financial sectors

every two years.

It examines how intensely the country’s legal and financial system allows wealthy

individuals and criminals to hide and launder money.

Parameters used in the ranking include automatic exchange of information and

registration of beneficial ownership.

According to TJN, an estimated $21 to 32 trillion of private financial wealth is

located, untaxed or lightly taxed, in secrecy jurisdictions (tax havens) around the

world.

A SERIES

97

SOURCE:

https://timesofindia.indiatimes.com/business/international-business/cayman-island-

ranks-first-on-the-financial-secrecy-index-with-switzerland-dropping-down-two-

ranks/articleshow/74211928.cms

74. Consider the following statements about a tiger reserve in India:

1. It is located in the North Eastern part of the country.

2. It is known for its Hornbill Nest Adoption Programme.

3. It adjoins Assam's Nameri National Park.

Which one of the following Tiger Reserve is described by the statements given

above?

(a) Dampa Tiger reserve

(b) Pakke Tiger Reserve

(c) Namdhapa Tiger Reserve

(d) Kamlang Wildlife Sanctuary

EXPLANATION:

Pakke Tiger Reserve, also known as Pakhui Tiger Reserve, is a Project Tiger reserve in the

Pakke Kessang district of Arunachal Pradesh in northeastern India. The reserve is

protected by the Department of Environment and Forest of Arunachal Pradesh.

It is bounded by Bhareli or Kameng River in the west and north, and by Pakke River in

the east. It is surrounded by contiguous forests on most sides. To the east lies Papum

Reserve Forest. Towards the south and south-east, the sanctuary adjoins reserve forests

and Assam's Nameri National Park. To the west, it is bounded by Doimara Reserve Forest

and Eaglenest Wildlife Sanctuary; and to the north by Shergaon Forest Division.

This Tiger Reserve has won India Biodiversity Award 2016 in the category of

'Conservation of threatened species' for its Hornbill Nest Adoption Program. So, option

(b) is correct.

ADDITIONAL INFORMATION:

Dampa Tiger Reserve

Dampa Tiger Reserve or Dampha Tiger Reserve is a tiger reserve of western Mizoram,

India. It covers an area of about 500 km2 in the Lushai Hills at an altitude range of

800–1,100 m (2,600–3,600 ft).

A SERIES

98

It was declared a tiger reserve in 1994 and is part of Project Tiger.

The tropical forests of Dampa Tiger Reserve are home to a diverse flora and fauna.

It consists of forest interpolated with steep precipitous hills, deep valleys, jungle

streams, ripping rivulets, natural salts licks. Dampa Tiger Reserve is not easily

accessible unlike other park where you can ride on a four wheeler but one has to

walk through the forest if one wishes to sight animals.

In the tiger census of 2018, no tiger was found in this reserve

Namdapha National Park

Namdapha National Park is a 1,985 km2 large protected area in Arunachal Pradesh

of Northeast India.

With more than 1,000 floral and about 1,400 faunal species, it is a biodiversity

hotspot in the Eastern Himalayas.

The national park harbours the northernmost lowland evergreen rainforests in the

world at 27°N latitude.

It also harbours extensive dipterocarp forests, comprising the northwestern parts of

the Mizoram-Manipur-Kachin rain forests ecoregion.

It is the fourth largest national park in India.

Kamlang Wildlife Sanctuary

The Kamlang Wildlife Sanctuary, established in 1989, is rich in flora and fauna.

It is situated in the Lohit District of the northeastern Indian state of Arunachal

Pradesh.

The park is named after the Kamlang River which flows through it.

The Mishmi, Digaru, and Mizo tribal people who reside around the periphery of the

sanctuary claim their descent from the King Rukmo of the epic Mahabharata.

They believe in a myth of an invisible god known as Suto Phenkhenynon jamalu.

An important body of water in the sanctuary is the Glow Lake.

Located in tropical and sub-tropical climatic zones, the sanctuary is the habitat of

the four big cat species of India: tiger, leopard, clouded leopard and snow leopard

75. With reference to Land Revenue system during the British India, which of the

following areas were associated with the “Permanent Settlement System”?

1. Bengal

2. Bihar

3. Banaras

4. Madras

A SERIES

99

5. Bombay

Select the correct answer using the code given below.

(a) 1 only

(b) 1, 2 and 3 only

(c) 4 and 5 only

(d) 1, 2, 3, 4 and 5

EXPLANATION:

The Ryotwari system was a land revenue system in British India, introduced by Thomas

Munro in 1820 based on system administered by Captain Alexander Read in the

Baramahal district. It allowed the government to deal directly with the cultivator ('ryot')

for revenue collection and gave the peasant freedom to give up or acquire new land for

cultivation. The peasant was assessed for only the lands he was cultivating. In Bombay,

Madras, Assam and Burma the Zamindar usually did not have a position as a middleman

between the government and the farmer. Hence, the Ryotwari system was followed in

these places.

The Permanent Settlement, also known as the Permanent Settlement of Bengal, was an

agreement between the East India Company and Bengali landlords to fix revenues to be

raised from land that had far-reaching consequences for both agricultural methods and

productivity in the entire British Empire and the political realities of the Indian

countryside.Revenues were collected by zamindars, native Indians who were treated as

landowners.The Permanent Settlement was followed in Bengal, Bihar, Varanasi and

Banaras. So, option (b) is correct.

76. Consider the following statements about the Question Hour of the Lok Sabha:

1. Generally, the second hour of a sitting of Lok Sabha is devoted to the Question

Hour.

2. Questions making disrespectful references to foreign countries with whom

India has friendly relations are not allowed.

3. A member of Lok Sabha is allowed to give not more than ten notices of

questions for a particular day.

Which of the statements given above are correct?

(a) 1 and 2 only

(b) 2 and 3 only

(c) 1 and 3 only

(d) 1, 2 and 3

A SERIES

100

EXPLANATION:

The first hour of every parliamentary sitting is allotted for this.So statement 1 is not

correct

Questions making disrespectful references to foreign countries with whom India has

friendly relations are not allowed. So statement 2 is correct

A member is allowed to give not more than ten notice of questions for a day. So

statement 3 is correct

ADDITIONAL INFORMATION:

Question Hour:

The first hour of every parliamentary sitting is allotted for this.

The concerned Minister is obliged to answer to the Parliament, either orally or in

writing, depending on the type of question raised.

Question Hour is not mentioned in the Constitution.

It finds mention in the Rules of Procedure of the House.

Types of Question:

Starred Questions:

These are Questions to which answers are desired to be given orally on the floor of

the House during the Question Hour.

These are distinguished in the printed lists by asterisks. 15 such questions are

listed each day.

Unstarred Questions:

These are Questions to which written answers are given by Ministers which are

deemed to have been laid on the Table of the House at the end of the Question Hour.

Upto 160 such questions are listed each day in a separate list.

Short Notice Questions:

Such questions can be asked orally in the House after the Question Hour or as the

first item in the agenda where there is no Question Hour at a notice shorter than

that prescribed for Starred and Unstarred Questions.

These must relate to a subject-matter considered by the Chairman to be of urgent

public importance.

A SERIES

101

SOURCE:

Indian Polity – Laxmikanth

77. Consider the following statement about the “Bio-Village”:

1. This concept was introduced in India based on the recommendation of M.S.

Swaminathan committee.

2. It is a geographical territory where the community people will practice only

organic farming where natural fertilizers and seeds could be used.

Which of the statements given above is/are correct?

(a) 1 only

(b) 2 only

(c) Both 1 and 2

(d) Neither 1 nor 2

EXPLANATION:

Bio-Village concept was not introduced in India based on the recommendation of M.S.

Swaminathan committee.So statement 1 is not correct.

Bio-village means a village/territory where community people would live and breathe in

nature and foster the biodiversified ecological balance. In this environment, they will

practice organic farming, where natural fertilizers and seed could be used.So statement

2 is correct.

ADDITIONAL INFORMATION:

The concept of 'bio-village', though of recent origin, has gained much success due to its

practical utility in addressing the issues faced by rural farmers across the nation. This

concept advocates adoption of eco-friendly technologies and enterprise development based

on biological resources, can be used as a tool in addressing the issues of sustainable

livelihood and food security. It provides the poor access to multiple sources of livelihood

through skill upgradation and technological empowerment. People in the states like Madhya

Pradesh, Rajasthan and Bihar have started to get benefit by adopting the concept of bio-

village. However, in the North Eastern region of the country it is yet to start materializing in

spite of the sincere efforts of organizations like National Institute of Rural Development.

This paper tries to highlight some of the sustainable technologies that can be undertaken

for the development of bio-villages in NE India which are carefully chosen keeping in mind

A SERIES

102

the inherent strengths like rich biodiversity and weaknesses like unfavourble terrain for

conventional agriculture. The technologies considered in detail here are the bio-briquette as

a source of clean energy and the trellises supplying the different nutritional requirements of

a family. These technologies, if adopted while developing bio-villages, would help to do away

with the problem of hunger and poverty to a great extent in the region especially in the

rural areas.

SOURCE:

https://economictimes.indiatimes.com/news/economy/agriculture/west-bengal-to-

go-organic-plans-to-set-up-bio-villages/articleshow/4332127.cms?from=mdr

78. Recently, Indian Air Force had test fired a beyond visual range air-air missile

named as “MICA”. Which of the following aircrafts that operates with Indian

Airforce is capable to fire “MICA” missile?

1. Mirage 2000

2. Rafale

3. Tejas

4. Sukhoi-30

Select the correct answer using the code given below.

(a) 1 and 2 only

(b) 2 and 3 only

(c) 1, 3 and 4 only

(d) 2, 3 and 4 only

EXPLANATION:

The MICA (Missile d’Interception, de Combat et d’Autodéfense) is a short and Beyond

Visual Range (BVR) AAM system developed by MBDA for the Rafale and advanced

variants of Mirage 2000 combat aircraft.

The system includes two variants, namely MICA (EM) RF featuring an active radio

frequency seeker and MICA IR featuring a dual waveband imaging infrared seeker to

defeat enemy countermeasures. A surface-launched version named VL MICA is also

available for use by naval or ground-based air defence systems.

The MICA is fitted with a HE warhead focused on splinter fragments and compatible with

any advanced fighter aircraft. Its lightweight and compact dimensions allow the

A SERIES

103

integration of up to six missiles on medium to lightweight fighters. So, option (a) is

correct.

79. Consider the following statements about a tribal community in India:

1. They are a tribal herdsmen community live predominantly in the state of

Gujarat.

2. They actively support the conservation of Asiatic Lions in India.

Which one of the following tribal community is indicated by the statements given

above?

(a) Meenas

(b) Gujjars

(c) Bhils

(d) Maldharis

EXPLANATION:

Maldharis are a tribal herdsmen community in Gujarat, India. Originally nomads, they

came to be known as Maldharis after settling in Junagadh district (mainly Gir Forest).

"Maldhari" is an occupational term which refers to people from a variety of castes and

communities. The literal meaning of Maldhari is keeper (dhari) of the animal stock (mal).

They are notable as the traditional dairymen of the region, and once supplied milk and

cheese to the palaces of rajas. In different regions, the Maldharis belong to different

castes. The Gir Forest National Park is home to around 8,400 Maldharis as of 2007. They

actively support the conservation of Asiatic Lions in India. So, option (d) is correct.

SOURCE:

https://www.thehindu.com/sci-tech/energy-and-environment/drought-in-a-desert-

why-has-drought-hit-the-maldharis-so-hard-this-year/article27090863.ece

80. With reference to the Medieval history of India, consider the following statements:

1. Mughal court chronicles were written in Turkish language.

2. All books in Mughal India were manuscripts.

A SERIES

104

Which of the statements given above is/are correct?

(a) 1 only

(b) 2 only

(c) Both 1 and 2

(d) Neither 1 nor 2

EXPLANATION:

Mughal court chronicles were written in Persian.As the Mughals were Chaghtai Turks by

origin, Turkish was heir mother tongue.It was Akbar who consciously set out to make

Persian the leading language of the Mughal court.So, statement 1 is not correct.

All books in Mughal India were manuscripts, that is, they were handwritten.Calligraphy,

the art of handwriting, was considered a skill of great importance.It was practised using

different styles. Akbar's favourite was the nastaliq, a fluid styte with long horizontal

strokes.So, statement 2 is correct.

ADDITIONAL INFORMATION:

Mughal court chronicles were written in Persian.

As the Mughals were Chaghtai Turks by origin, Turkish was heir mother tongue.

It was Akbar who consciously set out to make Persian the leading language of the

Mughal court.

Cultural and intellectual contacts with Iran, as well as a regular stream of Iranian

and Central Asian migrants seeking positions at the Mughal court, might have

motivated the emperor to adopt the language.

Mughal official histories such as the Akbar Nama were written in Persian, others,

like Babur's memoirs, were translated from the Turkish into the Persian Babur

Nama.

The Mahabharata was translated as the Razmnama (Book of Wars).

Painters too were involved in the production of Mughal manuscripts.

Chronicles narrating the events of a Mughal emperor's reign contained, alongside

the written text, images that described an event in visual form.

The historian Abu'l Fazi described painting as a "magical art", in his view it had the

power to make inanimate objects look as if they possessed life.

The production of paintings portraying the emperor, his court and the people who

were part of it, was a source of constant tension between rulers and representatives

of the Muslim orthodoxy, the ulama.

The latter did not fail to invoke the Islamic prohibition of the portrayal of human

beings enshrined in the Qur'an as well as the hadis.

A SERIES

105

Muslim rulers in many Asian regions during centuries of empire building regularly

commissioned artists to paint their portraits and scenes of life in their kingdoms.

81. Which one of the following country is not a part of Balkan Peninsula?

(a) Serbia

(b) Lithuania

(c) Albania

(d) Bulgaria

EXPLANATION:

So option (b) is correct.

82. Consider the following statements about the draft Battery Waste Management

Rules, 2020:

1. It will cover all types of batteries except lead-acid batteries.

2. It does not cover appliances into which a battery is, or maybe incorporated.

Which of the statements given above is/are correct?

(a) 1 only

(b) 2 only

(c) Both 1 and 2

(d) Neither 1 nor 2

A SERIES

106

EXPLANATION:

Draft Battery Waste Management Rules, 2020will cover all types of batteries including

lead-acid batteries. So, statement 1 is not correct.

The draft Rules will be applicable to various stakeholders involved in the life of batteries

or its components, consumables, and spare parts which make the product operational.

These include every manufacturer, producer, collection centre, importer, assembler,

dealer, recycler, consumer, and bulk consumers.So, statement 2 is not correct.

ADDITIONAL INFORMATION:

The Draft Rules seek to replace the Batteries (Management and Handling) Rules,

2001, which provide details for handling and management of batteries under the

Environment (Protection) Act, 1986. The 1986 Act regulates the protection and

improvement of the environment.

The draft Rules will be applicable to various stakeholders involved in the life of

batteries or its components, consumables, and spare parts which make the product

operational. These include every manufacturer, producer, collection centre,

importer, assembler, dealer, recycler, consumer, and bulk consumers.

Currently, the Batteries (Management and Handling) Rules, 2001, apply to only

lead-acid batteries.

The Draft Rules will cover all types of batteries. It will also apply to all appliances

into which a battery is, or maybe incorporated.

It will not apply to batteries used in certain equipment such as military equipment,

space exploration equipment, and emergency and alarm systems.

Responsibilities of manufacturer and dealers Under the Draft Rules:

Collecting used batteries against the new ones sold and issuing purchase invoices

(when they collect used batteries)

Setting up collection centres by themselves or jointly at various places for collecting

used batteries from dealers and consumers

Ensuring safe transport of the collected batteries to the authorized/registered

recyclers and

Filing an annual record of their sales and buyback to the state pollution control

board by December 31 of every year.

A SERIES

107

SOURCE:

https://theprint.in/environment/modi-govt-drafts-new-rules-on-battery-waste-

management-wants-people-to-give-suggestions/371391/

83. Consider the following statements about the “Uniform Civil Code” in India:

1. The original constitution of India did not contain the provision for making of

Uniform Civil Code in India.

2. Goa and Sikkim are the only Indian states to have a Uniform Civil Code in the

form of common family law.

Which of the statements given above is/are correct?

(a) 1 only

(b) 2 only

(c) Both 1 and 2

(d) Neither 1 nor 2

EXPLANATION:

Article 44 has provisions for uniform civil code. So statement 1 is not correct.

Goa has a common family law, thus being the only Indian state to have a uniform civil

code. The Special Marriage Act, 1954 permits any citizen to have a civil marriage outside

the realm of any specific religious personal law. Personal laws were first framed during

the British Raj, mainly for Hindu and Muslim citizens.So statement 2 is not correct

ADDITIONAL INFORMATION:

The Uniform Civil Code (UCC) calls for the formulation of one law for India, which would be

applicable to all religious communities in matters such as marriage, divorce, inheritance,

adoption. The code comes under Article 44 of the Constitution, which lays down that the

state shall endeavour to secure a Uniform Civil Code for the citizens throughout the

territory of India.

The issue has been at the center of political narrative and debate for over a century and a

priority agenda for the Bharatiya Janata Party (BJP) which has been pushing for the

legislation in Parliament. The saffron party was the first to promise the implementation of

UCC if it comes to power and the issue was part of its 2019 Lok Sabha election manifesto.

A SERIES

108

Why is Article 44 important?

The objective of Article 44 of the Directive Principles in the Indian Constitution was to

address the discrimination against vulnerable groups and harmonise diverse cultural

groups across the country. Dr. B R Ambedkar, while formulating the Constitution had said

that a UCC is desirable but for the moment it should remain voluntary, and thus the Article

35 of the draft Constitution was added as a part of the Directive Principles of the State

Policy in part IV of the Constitution of India as Article 44. It was incorporated in the

Constitution as an aspect that would be fulfilled when the nation would be ready to accept

it and the social acceptance to the UCC could be made.

Ambedkar in his speech in the Constituent Assembly had said, "No one need be

apprehensive that if the State has the power, the State will immediately proceed to

execute…that power in a manner may be found to be objectionable by the Muslims or by

the Christians or by any other community. I think it would be a mad government if it did

so."

Origin of Uniform Civil Code

The origin of the UCC dates back to colonial India when the British government submitted

its report in 1835 stressing the need for uniformity in the codification of Indian law relating

to crimes, evidence, and contracts, specifically recommending that personal laws of Hindus

and Muslims be kept outside such codification.

Increase in legislations dealing with personal issues in the far end of the British rule forced

the government to form the B N Rau Committee to codify Hindu law in 1941. The task of

the Hindu Law Committee was to examine the question of the necessity of common Hindu

laws. The committee, in accordance with scriptures, recommended a codified Hindu law,

which would give equal rights to women. The 1937 Act was reviewed and the committee

recommended a civil code of marriage and succession for Hindus.

What is the Hindu Code Bill?

The draft of the Rau Committee report was submitted to a select committee chaired by B R

Ambedkar that came up for discussion in 1951 after the adoption of the Constitution. While

discussions continued, the Hindu Code Bill lapsed and was resubmitted in 1952. The bill

was then adopted in 1956 as the Hindu Succession Act to amend and codify the law

relating to intestate or unwilled succession, among Hindus, Buddhists, Jains, and Sikhs.

A SERIES

109

The Act reformed the Hindu personal law and gave women greater property rights, and

ownership. It gave women property rights in their father's estate.

The general rules of succession under the Act 1956 for a male who dies intestate is that

heirs in Class I succeed in preference to heirs in other classes. An amendment to the Act in

the year 2005 added more descendants elevating females to Class I heirs. The daughter is

allotted the same share as is allotted to a son.

Difference between civil laws and criminal laws

While the criminal laws in India are uniform and applicable equally on all, no matter what

their religious beliefs are, the civil laws are influenced by faith. Swayed by religious texts,

the personal laws which come into effect in civil cases have always been implemented

according to constitutional norms.

What are personal laws?

Laws that apply to a certain group of people based on their religion, caste, faith, and belief

made after due consideration of customs and religious texts. The personal laws of Hindus

and Muslims find their source and authority in their religious ancient texts.

In Hinduism, personal laws are applicable to legal issues related to inheritance, succession,

marriage, adoption, co-parenting, obligations of sons to pay their father’s debts, the

partition of family property, maintenance, guardianship, and charitable donations. In Islam,

personal laws apply to matters relating to inheritance, wills, succession, legacies, marriage,

wakfs, dowry, guardianship, divorce, gifts, and pre-emption taking roots from Quran.

What will Uniform Civil Code do?

The UCC aims to provide protection to vulnerable sections as envisaged by Ambedkar

including women and religious minorities, while also promoting nationalistic fervour

through unity. When enacted the code will work to simplify laws that are segregated at

present on the basis of religious beliefs like the Hindu code bill, Shariat law, and others.

The code will simplify the complex laws around marriage ceremonies, inheritance,

succession, adoptions making them one for all. The same civil law will then be applicable

to all citizens irrespective of their faith.

A SERIES

110

SOURCE:

Indian Polity - Laxmikanth

84. Which one of the following type of natural vegetation has high shedding of its

leaves?

(a) Tropical evergreen forest

(b) Mid-latitude deciduous forest

(c) Steppe grassland

(d) Coniferous forest

EXPLANATION:

Tropical evergreen forest has high shedding of leaves. Though it appears evergreen all

trees shed their leaf in different parts of the years

ADDITIONAL INFORMATION:

Types of Natural Vegetation in India

(a) Tropical Evergreen Rain Forests

(b) Deciduous or Monsoon Type of Forests

(c) Dry Deciduous Forests

(d) Mountain Forests

(e) Tidal or Mangrove Forests

(f) Semi-Desert and Desert Vegetations

Tropical Evergreen Rain forests

The Tropical Evergreen rain forests are found in the areas where precipitation is more than

200 cm. They are largely found in the Northeastern regions of Arunachal Pradesh,

Meghalaya, Assam, Nagaland, the Western Ghats, the Tarai areas of the Himalayas and the

Andaman groups of Islands. They are also found in the hills of Khasi and Jaintia. The trees

in this area have intense growth. The major trees found in this area are Sandal Wood,

Rosewood, Garjan, Mahogany, and bamboo. It has copious vegetation of all kinds – trees,

shrubs, and creepers giving it a multilayered structure. The elephants, monkey, lemur are

the common animals found in these areas.

Deciduous or Monsoon type of forests

The Deciduous forests are found on the lower slope of the Himalayas, West Bengal,

Chhattisgarh, Bihar, Orissa, Karnataka, Maharashtra Jharkhand, and the adjoining

areas. The precipitation in this area is between 100 cm and 200 cm. Teak is the dominant

A SERIES

111

species seen in the area. Along with that Deodar, Blue Gum, Pal Ash, Sal, Sandalwood,

Ebony, Arjun, Khair, and Bamboo are also seen. The trees in this forest shed their leaves

during dry winter and dry summer. On the basis of the availability of water, these forests

are again divided into moist and dry deciduous.

Dry deciduous forests

These forests grow in areas where the precipitation is between 50 cm and 100 cm. These

are mainly seen in the areas of the Central Deccan plateau, Punjab, Haryana, parts of Uttar

Pradesh, Madhya Pradesh, and South-east of Rajasthan.

Mountain Forests/Montane Forests

Montane forests are those found in mountains. Mountain forests differ significantly along

the slopes of the mountain. On the foothills of the Himalayas until a height of 1500 meters,

evergreen trees like Sal, teak, and bamboo grow copiously. On the higher slope, temperate

conifer trees like pine, fir, and oak grow. At the higher elevation of the Himalayas,

rhododendrons and junipers are found. Further than these vegetation zones, alpine

grasslands appear up to the snowfield.

Tidal or Mangrove forests

The tidal or mangrove forests grow by the side of the coast and on the edges of the deltas

e.g., the deltas of the Cauvery, Krishna, Mahanadi, Godavari, and Ganga. In West Bengal,

these forests are known as ‘Sundarbans’. The ‘Sundari’ is the most major tree in these

forests. The important trees of the tidal forests are Hogla, Garan, Pasur, etc. This forest is

an important factor in the timber industry as they provide timber and firewood. Palm and

coconut trees beautify the coastal strip.

Semi-deserts and Deserts vegetations

This area receives a rainfall of less than 50 cm. Thorny bushes, acacia, and Babul are

found in this vegetation region. The Indian wild date is generally found here. They have long

roots and thick flesh. The plants found in this region store water in their stem to endure

during the drought. These vegetation are found in parts of Gujarat’s, Punjab, and in

Rajasthan.

SOURCE:

G C Leong

A SERIES

112

85. With reference to the philosophy of Jainism and Buddhism, consider the following

statements:

1. Both Jainism and Buddhism follow strict celibacy.

2. Both Buddhism and Jainism were against caste oppression.

3. Both Mahavira and Buddha felt that only those who left their homes could gain

true knowledge.

Which of the statements given above are correct?

(a) 1 and 2 only

(b) 2 and 3 only

(c) 1 and 3 only

(d) 1, 2 and 3

EXPLANATION:

Only Jainism followed strict celibacy and not buddism. So statement 1 is not correct.

Both Buddism and Jainism were against caste oppression. One of their main aim was

fighting against caste oppression. So statement 2 is correct.

Both the Mahavira and the Buddha felt that onlythose who left their homes could gain

trueknowledge.So statement 3 is correct.

ADDITIONAL INFORMATION:

Teachings of Jainism

Mahavira rejected Vedic principles.

He did not believe in God’s existence. According to him, the universe is a product of

the natural phenomenon of cause and effect.

He believed in Karma and transmigration of the soul. The body dies but the soul

does not.

One will be punished or rewarded as per one’s karma.

Advocated a life of austerity and non-violence.

Stressed on equality but did not reject the caste system, unlike Buddhism. But he

also said that man may be ‘good’ or ‘bad’ as per his actions and not birth.

Asceticism was taken to a great length. Starvation, nudity, and self-mortification

were expounded.

Two elements of the world: Jiva (conscious) and Atma (unconscious):

A SERIES

113

Right faith

Right knowledge

Right conduct (observance of five vows)

1. Ahimsa (non-violence)

2. Satya (truth)

3. Asteya (no stealing)

4. Parigraha (no acquiring property)

5. Brahmacharya (abstinence)

Teachings of the Buddha

The teaching are mentioned below:

It teaches the Middle Path renouncing extreme steps like indulgence and strict

abstinence.

The four noble truths (Arya Satya) in Buddhism are:

1. The world is full of sorrow

2. Desire is the root cause of all sorrow

3. Sorrow can be conquered by conquering desire

4. Desire can be conquered by following the eight-fold paths (Ashtangirka Marga)

The eight-fold path in Buddhism is:

Right understanding

Right resolve

Right speech

Right action

A SERIES

114

Right living

Right efforts

Right thought

Right self-concentration.

The Tri Ratnas of Buddhism are: Buddha, Dhamma and Sangha. Details are

mentioned below:

Buddha: the highest spiritual potential in everyone.

Dhamma: the teachings of Buddha (Pali for Sanskrit Dharma or righteousness)

Sangha: order of monks who follow Buddhism.

Buddha did not believe in God or the soul.

Stressed on karma and ahimsa.

He was against the varna system. Buddha taught in Pali.

Buddhism spread to several countries outside India. China adopted Buddhism in

the 1st century AD. [/su_box]

SOURCE:

History ncert

86. The term “phantom capital”, sometimes seen in the news recently, is

(a) a specified amount of capital that banks are required to hold as a risk mitigation

requirement

(b) the capital typically passes through empty corporate shells that have no real

business activity

(c) the capital invested for ongoing production of goods and services with aim of

maximising profit

(d) the capital and financial assets that Individuals hold as part of their net worth

A SERIES

115

EXPLANATION:

The Phantom capitalis phantom in nature – investments that pass through empty

corporate shells.These shells, also called special purpose entities, have no real business

activities. Rather, they carry out holding activities, conduct intrafirm financing, or

manage intangible assets – often to minimise multinationals’ global tax bill.So, option (b)

is correct.

ADDITIONAL INFORMATION:

About $15 trillion, or 38 per cent, of the world’s foreign direct investment (FDI) in

2017 was “phantom capital” that was tailor-made to trim tax bills of multinational

corporations, and tax havens were being used to funnel these investments,

according a study put out by the International Monetary Fund.

The share of such investments in reported FDI has shot up by roughly seven

percentage points since 2010 and serves to skew the global FDI data, shows the

study by the International Monetary Fund (IMF) and the University of Copenhagen.

Roughly a half of the phantom FDI — “investments that pass through empty

corporate shells” with no real business activity — passes through just Luxembourg

and the Netherlands.

Once Hong Kong, the British Virgin Islands, Bermuda, Singapore, the Cayman

Islands, Switzerland, Ireland, and Mauritius are added to the list, these 10

economies host more than 85 per cent of all phantom investments, the study finds.

These nations typically have low tax rates.

“FDI is often an important driver for genuine international economic integration,

stimulating growth and job creation and boosting productivity,” the report said.

However, phantom capital is typically “financial and tax engineering” that “blurs

traditional FDI statistics and makes it difficult to understand genuine economic

integration”.

SOURCE:

https://indianexpress.com/article/business/rise-in-phantom-capital-skews-global-fdi-

data-5984560/

87. Which one of the following cities experience Trade winds at least for once a year?

(a) San Francisco

(b) Reykjavik

A SERIES

116

(c) Beijing

(d) Ottawa

EXPLANATION:

88. Which of the following is/are the consequence(s) of Swadeshi movement which

took place during the early part of the twentieth century in India?

1. The British followed the ‘policy of repression’ over the students who supported

the movement to refrain them from joining national schools and colleges.

2. Prosecutions against a large number of nationalist newspapers were launched

and freedom of the press was completely suppressed.

Which of the statements given above is/are correct?

(a) 1 only

(b) 2 only

(c) Both 1 and 2

(d) Neither 1 nor 2

EXPLANATION:

During the Swadeshi movement, police adopted a repressive attitude towards

thestudents. Schools and colleges whose students participatedin the agitation were to be

penalised by disaffiliating themor stopping of grants and privileges to them. Students

whowere found guilty of participation were to be disqualified forgovernment jobs or for

government scholarships, anddisciplinary action—fine, expulsion, arrest, beating, etc.—

was to be taken against them.So, statement 1 is correct.

A SERIES

117

Prosecutions against a large number of nationalist newspapers were launched and

freedom of the press was completely suppressed. So, statement 2 is correct.

ADDITIONAL INFORMATION:

Swadeshi Movement was started against partition and got formal proclamation of

the movement was on August 7, 1905 at a meeting held at the Calcutta Town hall.

It was suggested by Krishan Kumar Mitra’s journal Sanjivani in AD 1905. In this

movement, Swadeshi leaders appeal to Indian for boycotting of government service,

courts, schools and colleges and of foreign goods, promotion of Swadeshi goods,

Promotion of National Education through the establishment of national schools and

colleges. Hence, it was not only political but economic movement as well.

The Swadeshi Movement was a great success. In Bengal, even the landlords joined

the movement. The women and students took to picketing. Students refused using

books made of foreign paper.

Many leaders were imprisoned and deported like Bal Gangadhar Tilak, Lala Lajpat

Rai, Bipin Chandra Pal and Aurobindo Ghosh. Many Indian lost their job and

student who took part in movement were not allowed to enter college and school

premises.

During the movement, singing Vande Mataram was meant to sedition. This was the

first instance in which use of Indigenous goods were taken an account.

SOURCE:

Old NCERT History Modern India by Bipin Chandra (Class 12)

89. Consider the following statements:

1. Continental Shelf of Western Coast of India is larger than Eastern Coast.

2. On an average Arabian sea has lower temperature compared to Bay of Bengal.

3. On an average Arabian sea has high salinity than Bay of Bengal.

Which of the statements given above is/are correct?

(a) 1 only

(b) 2 only

(c) 1 and 2 only

(d) 1, 2 and 3

A SERIES

118

90. Consider the following statements about the Minimum Alternate Tax in India:

1. It is applicable for both corporate entities and non-corporate entities operating

in India.

2. It is not required to be paid by foreign companies having no Permanent

Establishments in the country.

Which of the statements given above is/are correct?

(a) 1 only

(b) 2 only

(c) Both 1 and 2

(d) Neither 1 nor 2

EXPLANATION:

Minimum Alternate Tax is applicable to all corporate entities, whether public or private.

However, it does not apply to any income accruing or arising to a company from life

insurance business. The corresponding tax similar to MAT, but imposed on individuals or

non-corporate entities, who claim certain deductions under the IT Act (deduction under

section 80H to 80RRB (except 80P), deduction under section 35AD and deduction under

section 10AA), is known as Alternate Minimum Tax (AMT).So, statement 1 is not

correct.

MAT would be applicable only to those foreign companies which have permanent

establishment in India. Based on these rulings, a view was being taken that MAT did not

apply to foreign companies not having permanent establishment in India. So, statement

2 is correct.

ADDITIONAL INFORMATION:

Minimum Alternate Tax (MAT) is a tax effectively introduced in India by the Finance

Act of 1987, vide Section 115J of the Income Tax Act, 1961 (IT Act), to facilitate the

taxation of ‘zero tax companies’ i.e., those companies which show zero or negligible

income to avoid tax. Under MAT, such companies are made liable to pay to the

government, by deeming a certain percentage of their book profit as taxable income.

MAT is an attempt to reduce tax avoidance; it was introduced to contain the

practices followed by certain companies to avoid the payment of income tax, even

though they had the “ability to pay”.

MAT is applied when the taxable income calculated as per the normal provisions in

the IT Act is found to be less than 18.5% of the book profits.

MAT is levied at the rate of 18.5% of the book profits. MAT rate has been

progressively increased from 7.5% in 2000 to 18.5% in 2015. In other words, the tax

A SERIES

119

computed by applying 18.5% (plus surcharge and cess as applicable) on book profit

is called MAT.

Normal tax rate applicable to an Indian company is 30% (plus cess and surcharge as

applicable), which has been decided to be progressively reduced to 25% by 2019. A

company has to pay higher of normal tax liability or liability as per MAT provisions.

MAT is applicable to all corporate entities, whether public or private. However, it

does not apply to any income accruing or arising to a company from life insurance

business. Nor does it apply to shipping income liable to tonnage taxation[2] as

provided in section 115V to 115VZC of the IT Act.

The corresponding tax similar to MAT, but imposed on individuals or non-corporate

entities, who claim certain deductions under the IT Act (deduction under section

80H to 80RRB (except 80P), deduction under section 35AD and deduction under

section 10AA), is known as Alternate Minimum Tax (AMT). The rate of AMT is also at

18.5%.

SOURCE:

http://www.arthapedia.in/index.php?title=Minimum_Alternate_Tax_(MAT)

91. Which one of the following best describes the term “Social banking”?

(a) Extension of Credit to young demography

(b) Providing loans for the skill development initiatives

(c) Providing banking services for the poor population of the country

(d) Providing loans for the health and education sector

92. Which one of the following Ashoka inscription speaks about the success of

Ashoka’s policy with hunters and fisherman who gave up killing animals?

(a) Kandahar inscription

(b) Mansehra inscription

(c) Lampaka inscription

(d) Shahbazgarhi inscription

EXPLANATION:

Kandhar inscription of Ashoka is a famous bilingual edicts in Greek and Aramaic,

proclaimed and craved in stone by the Indian Mauryan Empire ruler Ashoka. It was

discovered in 1958. It is an inscription of Ashoka that tells the success of his policy with

the hunters and fishermen, who gave up killing animals and took to settled agricultural

A SERIES

120

life.So, option (a) is correct.

ADDITIONAL INFORMATION:

Mansehra inscription

Mansehra Rock Edicts are fourteen edicts of the Mauryan emperor Ashoka,

inscribed on rocks in Mansehra in Khyber Pakhtunkhwa, Pakistan.

The edicts are cut into three boulders and date back to 3rd century BC and they are

written in the ancient Indic script of Gandhara culture, Kharosthi. The edicts

mention aspects of Ashoka’s dharma.

The site was submitted for inclusion in the World Heritage Sites and is currently in

the tentative list.

Lampaka inscription

The Pul-i-Darunteh Aramaic inscription, also called Aramaic inscription of Lampaka,

is an inscription on a rock in the valley of Laghman ("Lampaka" being the

transcription in Sanskrit of "Laghman"), Afghanistan, written in Aramaic by the

Indian emperor Ashoka around 260 BCE.

It was discovered in 1932 at a place called Pul-i-Darunteh.

Since Aramaic was the official language of the Achaemenid Empire, which

disappeared in 320 BCE with the conquests of Alexander the Great, it seems that

this inscription was addressed directly to the populations of this ancient empire still

present in northwestern India, or to border populations for whom Aramaic remained

the language of use.

Shahbazgarhi inscription

The Shahbazgarhi rock edicts are cut into the surface of two large boulders on the

side of a small rocky outcrop in the Vale of Peshawar. The record fourteen edicts of

the Mauryan emperor, Asoka (r. c. 272-235 BC) and represent the earliest

irrefutable evidence of writing in South Asia.

Dating to middle of the third century BC, they are written from right to left in the

Kharosthi script.

The presence of Kharosti suggests that the influence of Achaemenid rule in this

region, the province of Gandhara, outlived the short Alexandrian interlude of the

fourth century BC.

The fourteen major edicts recorded at the site present aspects of Asoka’s dharma or

righteous law. The edicts are located beside one of the ancient trade routes

A SERIES

121

connecting the Vale of Peshawar with the valley of Swat, Dir and Chitral to the North

and the great city of Taxila to the South East.

93. Consider the following statements about the “Unlawful Activities (Prevention) Act,

1967”:

1. This Act empowers the government to declare an organisation as “terrorist”

and ban it within the territory of India.

2. The ‘unlawful activities’, defined by this act, includes terrorism and terror

financing but excludes money laundering.

Which of the statements given above is/are correct?

(a) 1 only

(b) 2 only

(c) Both 1 and 2

(d) Neither 1 nor 2

EXPLANATION:

Under the Act, the central government may designate an organisation as a terrorist

organisation if it:

o commits or participates in acts of terrorism

o prepares for terrorism

o promotes terrorism

o is otherwise involved in terrorism

so statement 1 is correct.

The unlawful activities include terrorism and terror financing and also money laundering

so statement 2 is not correct.

ADDITIONAL INFORMATION:

What is the Act on?

The Act amends the Unlawful Activities (Prevention) Act, 1967 (UAPA).

The original Act dealt with “unlawful” acts related to secession; anti-terror

provisions were introduced in 2004.

It provides special procedures to deal with terrorist activities, among other things.

Concern - There is widespread opposition to the amendments on the ground that it

could be used to target dissent against the government.

The provisions could potentially affect citizens’ civil rights.

A SERIES

122

What are the key provisions in the Act?

Definition - Under the Act, the central government may designate an organisation as

a terrorist organisation if it:

o commits or participates in acts of terrorism

o prepares for terrorism

o promotes terrorism

o is otherwise involved in terrorism

The Act additionally empowers the government to designate individuals as terrorists

on the same grounds.

The word “terror” or “terrorist” is not defined.

However, a “terrorist act” is defined as any act committed with the intent -

to threaten or likely to threaten the unity, integrity, security, economic security, or

sovereignty of India

to strike terror or likely to strike terror in the people or any section of the people in

India or in any foreign country

Investigation by NIA - Under the Act, investigation of cases may be conducted by

officers of the rank of Deputy Superintendent or Assistant Commissioner of Police or

above.

The Act additionally empowers the officers of the National Investigation Agency (NIA),

of the rank of Inspector or above, to investigate cases.

Seizure of property by NIA - Under the Act, an investigating officer can seize

properties that may be connected with terrorism with prior approval of the Director

General of Police.

The amendment Act, however, removes this requirement if the investigation is

conducted by an officer of the NIA.

The investigating officer, in that case, only requires sanction from the Director

General of NIA.

[Central agencies such as the CBI are required to obtain prior permission from the

state government since law and order is a state subject under the Constitution.]

Insertion to schedule of treaties - The Act defines terrorist acts to include acts

committed within the scope of any of the treaties listed in a schedule to the Act.

The Schedule lists 9 treaties, including -

o the Convention for the Suppression of Terrorist Bombings (1997)

o the Convention against Taking of Hostages (1979)

The Act adds another treaty to the list, which is the International Convention for

Suppression of Acts of Nuclear Terrorism (2005).

A SERIES

123

Designation - The central government may designate an individual as a terrorist

through a notification in the official gazette.

His/her name is added to the schedule supplemented to the UAPA Act.

The government is not required to give an individual an opportunity to be heard

before such a designation.

At present, legally, a person is presumed to be innocent until proven guilty.

In this line, an individual who is convicted in a terror case is legally referred to as a

‘terrorist’.

And those suspected of being involved in terrorist activities are referred to as ‘terror

accused’.

The Act does not clarify the standard of proof required to establish that an

individual is involved or is likely to be involved in terrorist activities.

On designation - The designation of an individual as a ‘global terrorist’ by the United

Nations is associated with sanctions.

The UAPA Act, however, does not provide any such detail.

The Act also does not require the filing of cases or arresting individuals while

designating them as terrorists.

The consequences of the designation will be prescribed in the Rules supplemented to

the law once the amendment Act is passed.

How can the names be removed?

Application - The Act seeks to give the central government the power to remove a

name from the schedule when an individual makes an application.

The procedure for such an application and the process of decision-making will also

be decided by the central government.

If an application filed is rejected by the government, the Act gives the person the

right to seek a review within one month of rejection.

Review committee - Under the amendment Act, the central government will set up a

review committee.

It will consist of a chairperson (a retired or sitting judge of a High Court) and 3 other

members.

It will be empowered to order the government to delete the name of an individual

from the schedule that lists “terrorists”, if it considers the order to be flawed.

Apart from these two avenues, the individual can also move the courts challenging

the government’s order.

A SERIES

124

SOURCE:

https://www.iasparliament.com/current-affairs/unlawful-activities-prevention-

amendment-bill-

2019#:~:text=The%20Unlawful%20Activities%20Prevention%20Amendment,provision

s%20were%20introduced%20in%202004.

94. With reference to the political organizations during the British era, who among

the following were the founders of the “Bombay Association” in 1852?

(a) Pheroshah Mehta, Badruddin Tayabji and KT Telang

(b) Jaggannath Shankersheth, Naoroji Fursungi and Dadabhai Naoroji

(c) Anand Mohan Bose, Surendranath Banerjea and Sisir Kumar Ghosh

(d) MahadevGovind Ranade, S.H. Chiplunkar and Ganesh Vasudeo Joshi

95. Consider the following water bodies:

1. Lake Baikal

2. Dead Sea

3. River Rhine

Which of the water bodies mentioned above is/are present in a Rift Valley?

(a) 1 only

(b) 1 and 2 only

(c) 2 and 3 only

(d) 1, 2 and 3

EXPLANATION:

The Baikal Rift Zone is a series of continental rifts centered beneath Lake Baikal in

southeastern Russia.So statement 1 is correct.

The Dead Sea is a rift lake in the Jordan Rift Valley. Although the Dead Sea is not the

world's deepest lake, the deep Jordan Rift makes it the lowest land elevation on Earth.So

statement 2 is correct.

The Upper Rhine Graben formed during the Oligocene as a response to the evolution of

the Alps to the south and remains active to the present day. Today, the Rhine Rift

Valley forms a downfaulted trough through which the river Rhine flows.So statement 3

is correct.

A SERIES

125

ADDITIONAL INFORMATION:

Africa is splitting

A place near Nairobi, Kenya had a large crack on the ground that appeared during

an intense spell of rain and flooding.

The crack is 57 km long which is a locus of a future ocean formation.

Such an event happened 138 million years ago when the South American and

African continents separated to give rise to the current South Atlantic Ocean.

All such signs on Earth’s surface point to a totally new landscape, which resembled

how it looked like when it was formed 4.5 billion years ago one huge continent.

The farthest scientists can predict is that 250 million years from now all continents

will unite again in one supercontinent and then will break apart again along the

future rift systems.

Why such cracks?

At first, geologists thought the crack in Kenya was formed due to “erosion of soft

soils infilling an old rift-related fault.

They later revealed that the crack had existed for quite some time, but was filled

with ash from Mount Longmont, a volcano nearby.

The rain had washed away the ash to expose the crack. This triggered a debate

whether the crack was a part of the East African Rift system.

Rift in systems

Rifts are the regions of extension of the crust and the lithosphere.

Continental changes take place at the boundaries of tectonic plates which are

divisions of the uppermost layer of the Earth and swim around on the fluid mantle

layer below it.

The extension may develop to a stage when two plates split apart, like in the

example of the Red Sea and the Gulf of Aden. So continental rifts are potential

places where new oceans are expected to form.

These plates periodically crash into each other, giving rise to earthquakes and

volcanic eruptions when the intensity is less over short periods of time.

Over longer periods of time, they create all the geological and geographical features

that we find on Earth like mountains, valleys and oceans.

Intra-continental drifting

One of these processes is an intra-continental rift system which acts between

tectonic plates and can give rise to rift valleys or even new oceans.

A SERIES

126

The African Rift Valley, which is between Ethiopia and Kenya, is a classical example

of this geodynamic process.

There, volcanism, earth-quakes and fracturing of the Earth’s surface result from the

enormous forces that tear the eastern portion of the African continent apart.

East African rift system

The East African rift system is more active in terms of volcanism and it is connected

to the global ocean rift (ridge) system through the Afar-Red Sea—Gulf of Aden triple

junction.

The Red Sea and the Gulf of Aden are the rifts along which two continents break

apart and new oceanic flow is formed.

There are numerous rift systems around the world but the most active ones are the

East African rift, Baikal rift, West Antarctic rift, Rio Grande rift, the Rhine Graben

rift system in Europe and Shanxi rift system in China.

When such geographical features become prominent enough they reshape the way

the planet looks.

The rifts undergo massive geological changes—shoulders of rifts grow and get eroded

by rain and melting snow.

At the same time, the axial parts of rifts subside and get filled with sediments, which

gets eroded from the shoulders.

SOURCE:

G C Leong

96. Consider the following climatic types:

1. Steppe

2. Laurentian

3. Siberian

Which of the climatic type(s) mentioned above is/are found in Russia?

(a) 3 only

(b) 1 and 2 only

(c) 2 and 3 only

(d) 1, 2 and 3

A SERIES

127

EXPLANATION:

So all three climates are found in Russia. So option (d) is correct

ADDITIONAL INFORMATION:

Steppe Climate

Temperature

Climate is continental with extremes of temperature.

Temperatures vary greatly between summer and winter.

The summers are hot and the winters are cold.

Summers are very warm, over 18 – 20° C.

The steppe type of climate in the southern hemisphere is never severe.

A SERIES

128

Precipitation

The average rainfall may be taken as about 45 cm, but this varies according to

location from 25 cm to 75 cm.

The heaviest rain comes in June and July (late spring and early summer).

Most of the winter months have about an 2.5 cm of precipitation, brought by the

occasional depressions of the Westerlies and coming in the form of snow.

The maritime influence in the southern hemisphere causes more rainfall.

Natural Vegetation of Steppe Climate

Grasses

Greatest difference from the tropical savanna is that steppes are

practically treeless and the grasses are much shorter.

Grasses are tall, fresh and nutritious. This is typical of the grass of the wheat-lands

in North America, the rich black earth or chernozem areas of Russian Ukraine and

the better watered areas of the Asiatic Steppes.

Where the rainfall is light or unreliable, or the soil is poor, as in the continental

interiors of Asia the short steppe type of grass prevails.

The grasses are not only shorter but also wiry [lean, tough] and sparse [thinly

dispersed or scattered].

These areas are less suitable for arable farming and are used for some form

of ranching as in the High Plains of U.S.A.

The growth of grasses is not abruptly checked by summer droughts or winter cold.

Trees

Polewards, an increase in precipitation gives rise to a transitional zone of wooded

steppes where some conifers gradually appear.

In the cultivated regions, such as the wheat farms of the Prairies, double rows of

trees are planted around the house to shield the occupants from the strong wind.

Animals

Does not have much animal diversity.

Horses are common in Asian Steppes.

Laurentian climate i. The North American Region:

The most remarkable characteristic of the Laurentian climate of the North American region

is its uniformity in precipitation (about 3 to 4 inches monthly) with a late summer

A SERIES

129

maximum. New York in Fig. 154(a) has an annual precipitation of 41 inches with the

wettest months in July and August (4.2 inches each).

No month is really dry, and the driest month, November, has 2.5 inches of rain. This

uniformity of precipitation is largely due to the Atlantic influence and that of the Great

Lakes. The warm Gulf Stream increases the moisture content of easterly winds from the

open Atlantic.

The prevailing Westerlies which penetrate across the Rockies carry depressions over the

Great Lakes to the New England states. These winds thus promote wet conditions especially

in winter, which are vital for the agricultural activities of northeastern North America.

The meeting of the warm Gulf Stream and the cold Labrador Current on coastal waters off

Newfoundland produces dense mist and fog and gives rise to much precipitation.

St. John’s, its capital has as much as 54 inches of annual precipitation. It is said that

Newfoundland experiences more drizzles than any other part of the world. In summer the

Westerlies bring less depression and extend their continental influence to the coast.

Temperatures are normally high in summer for the latitude.

New York has a mean July temperature of 74°F. and sometimes even as high as 90°F. Once,

on 7 August 1918, the absolute maximum of 104°F. was reached.

Such high temperatures in a cool temperate maritime region, where the relative humidity is

high, can be very trying. Prolonged heat waves cause discomfort and frustration in crowded

cities. In winter, the temperature drops and snow falls.

New York has two months below freezing- point, and an annual temperature range of 43°F.

Away from the maritime influence, the cold increases. The mean January temperatures for

Quebec, Ottawa and Montreal are 10°F., 12°F. and 14°F respectively. The temperature

ranges widen accordingly.

ii. The Asiatic Region:

In contrast, the rainfall distribution of the Asiatic region is far less uniform. Winters are

cold and very dry while summers are very warm and exceptionally wet. Peking, a typical

station of the Laurentian Climate in northern China will bring out these facts very clearly.

It has seven dry months from October to April with a total rainfall of less than 2.1 inches

which is only one- twelfth of the annual total of 25 inches. The remaining five months

receive more than an inch a month, with 9.4 inches in July alone.

A SERIES

130

The rainfall regime is, in fact, similar to that of the tropical monsoon type in India, where

the whole year’s rainfall is concentrated in the three summer months. The mountainous

interior of China has such pronounced continental effects that the intense heating in

summer creates a region of extreme low pressure, and moisture-laden winds from the

Pacific Ocean and the Sea of Japan blow in as the South-East Monsoon.

The Laurentian type of climate here is often described as the Cool Temperate Monsoon

Climate. It has a very long, cold winter, and a big annual range of temperature. The July

mean for Peking is 79°F. while that of January is only 24°F.

The temperature range is therefore more than 55°F. The dry, cold wind that blows out from

the heart of Asia in winter carries fine, yellowish dust and deposits it as a thick mantle of

loess in Shansi, Shensi and other neighbouring provinces at the bend of the Hwang Ho.

Much of the winter precipitation in northern China, Korea and Hokkaido, Japan, is in the

form of snow. In the mountainous districts, the snow piles to a depth of 5 to 10 feet. The

climate of Japan is modified by its insularity, and also by the meeting of warm and cold

ocean currents. It receives adequate rainfall from both the South-East Monsoon in summer

and the North- West Monsoon in winter.

The latter is the dry, cold wind from mainland Asia, but after crossing the Sea of Japan it

has gathered sufficient moisture to give heavy relief rain or snow on the western coasts of

Japan. The rainfall is more evenly distributed, as in Tokyo, with two maxima, one in June,

the Plum Rain (6.5 inches), and the other in September, the Typhoon Rain (9 inches).

On the windward slopes of the Japanese Alps on the west, some stations, e.g. Kanazawa,

have more than 102 inches of rain, much of it falling as snow. The maritime influence also

effectively moderates the temperature range.

Tokyo has a range of 40°F. (79°F. in August and 39°F. in January) with none of the months

below freezing-point. The warm Kuroshio has played an important part in making the

climate of Japan less extreme. In meeting the cold Oyashio from the north, it also produces

fog and mist, making north Japan a ‘second Newfoundland’. Fishing replaces agriculture as

the main occupation in many of the indented coastlands.

Natural Vegetation of Laurentian Type of Climate:

The predominant vegetation of the Laurentian type of climate is cool temperate forest. The

heavy rainfall, the warm summers and the damp air from fogs, all favour the growth of

trees. Generally speaking, the forest-tend to be coniferous north the 50°N. parallel of

latitude.

A SERIES

131

The increase in the length and severity of the winter excludes fore that are not adaptable to

cold conditions. In the Asiatic region (eastern Siberia and Korea), the coniferous forests are,

in fact, a continuation of the great coniferous belt of the taiga. Lumbering has always been

a major occupation of this sparsely populated part of eastern Asia and timber is a leading

export item.

Much of the original coniferous forest of fir, spruce and larch has been cleared as a result of

lumbering rather than agriculture. Eastern Canada, along the banks of the St. Lawrence

River is the heart of the Canadian timber and wood pulp industry.

South of latitude 50°N., the coniferous forest give way to deciduous forests. Oak, beech,

maple and birch are the principal trees. Like their counterparts on the western margins, the

deciduous forests are fairly open. A long growing season of over six months and an

adequate supply of moisture from maritime sources encourage rapid growth of ferns and

other temperate undergrowth.

The occurrence of trees in almost pure stands, and the predominance of only a handful of

species greatly enhance the commercial value of these forests. As a result, they have been

extensively felled for the extraction of temperate hardwood.

In many parts of Manchuria, Korea and Japan, the forests have made way for the plough.

Both food and cash crops are raised. In Canada, due to the greater reserves of coniferous

softwoods and their overriding importance in industrial uses, the annual production of

deciduous hardwood is much less significant

Siberian climate

Siberian type climate is typified by bitterly cold winters of long duration & cool brief

summer; lies across 60* N of equator.

Spring & autumn are merely brief transitional periods

Annual range of temperature is quite high due to extremes of temperature observed

in this type of climate, as temp. well below freezing point in winters & approx. 15* in

summers.

With low temperatures in cold season, heavy snowfall can be expected, with frost

occurring as early as August

By September, most of the lakes & ponds are icebound; with the number of days in

which the rivers are frozen, increases from south to north.

The interiors of Eurasian continent are so remote from maritime influence that

annual precipitation cannot be high.

A SERIES

132

Generally, a total of 40 – 60 cm of rainfall is typified in this sub-arctic type of

climate.

Rainfall is quite well distributed throughout the year, with a summer maximum

from the convectional rain, when the continental interiors are greatly heated approx.

20* C.

The total precipitation of Siberian climate is marked by many factors viz. altitude,

latitude, proximity to the poles, amount of exposure to influences by westerlies (on

western part of the continent), temperate monsoon (on eastern part of the continent)

& penetration of cyclones.

Snow falls nearly everywhere in USSR in long, cold winter, but the amount varies

from place to place; with heaviest in northern tundra & Siberian taiga.

Permanent snowfields like of Alps or the Himalayas are absent, because any

accumulation of snow is melted with the return of spring & the warm summer.

Frozen rivers are thawed, causing a rise in the water level, sometimes resulting in

extensive floods.

Being a poor conductor of heat, the presence of thick mantle of snow protects the

soil of ground from severe cold above, which may be approx. 5* – 10* C colder.

It also provides moisture for the vegetation when the snow melts in spring

Natural Vegetation

No other trees are as well adapted as the conifers, to withstand such a severe

inhospitable environment as Siberian type of climate.

Coniferous belts of Eurasia & North America are the richest sources of softwood;

Used in construction, furniture, matches, paper & pulp, rayon & other chemical

products.

The world’s greatest softwood producers are USSR, USA, Canada & Fenoscandian

countries (Finland, Sweden & Norway).

USA is the leading producer in the production of wood pulp & Canada in newsprint,

accounting for almost half of the world’s production.

SOURCE: G C Leong

97. The term “Kr00k”, sometimes seen in the news recently, is a

(a) method to avoid train collision during extreme fog accumulation

(b) vulnerability that impacts Wi-Fi communications

A SERIES

133

(c) zoonotic disease that got widely spread in the Asian countries

(d) large scale air purifier that was developed by CSIR

EXPLANATION:

Kr00k is a security vulnerability that allows some WPA2 encrypted WiFi traffic to be

decrypted. The vulnerability was originally discovered by security company ESET in

2019. ESET estimates that this vulnerability affects over a billion devices. So, option (b)

is correct.

ADDITIONAL INFORMATION:

ESET, a Slovak cybersecurity firm, has for the last one year been working in

collaboration with manufacturers of two specific WiFi chips, which were found to

have been vulnerable to attack by hostile parties.

The manufacturers have since released for all the vulnerable devices security

patches that users can install to address the vulnerability.

According to a report released by ESET, the vulnerability named Kr00k was

discovered while the firm was conducting research into KRACK, a series of attacks

on WiFi devices that was detected last year.

In subsequent investigations, ESET discovered that WiFI chips manufactured by

Broadcom ad Cypress had the Kr00k vulnerability. Chips from both these

companies have a high market share and are used in a wide range of WiFi-enabled

devices.

SOURCE:

https://www.thehindu.com/sci-tech/technology/slovak-firm-spots-vulnerability-in-

16-devices/article30965469.ece

98. With reference to the Constitution of Inida, consider the following statements

about the “Fundamental Duties” of an Indian citizen:

1. Fundamental Duties can be enforced only through writs.

2. The original constitution of India did not contain fundamental duties.

Which of the statements given above is/are correct?

(a) 1 only

(b) 2 only

A SERIES

134

(c) Both 1 and 2

(d) Neither 1 nor 2

EXPLANATION:

Fundamental Duties can be enforced by through legislations. So statement 1 is not

correct.

As of now, there are 11 Fundamental duties. Originally, the Constitution of

India did not contain these duties. Fundamental duties were added by 42nd and

86th Constitutional Amendment acts. So statement 2 is correct.

ADDITIONAL INFORMATION:

Article 51A: Fundamental duties

It shall be the duty of every citizen of India –

(a) to abide by the Constitution and respect its ideals and institutions, the National Flag

and the National Anthem;

(b) to cherish and follow the noble ideals which inspired our national struggle for freedom;

(c) to uphold and protect the sovereignty, unity and integrity of India;

(d) to defend the country and render national service when called upon to do so;

(e) to promote harmony and the spirit of common brotherhood amongst all the people of

India transcending religious, linguistic and regional or sectional diversities; to renounce

practices derogatory to the dignity of women;

(f) to value and preserve the rich heritage of our composite culture;

(g) to protect and improve the natural environment including forests, lakes, rivers and wild

life, and to have compassion for living creatures;

(h) to develop the scientific temper, humanism and the spirit of inquiry and reform;

(i) to safeguard public property and to abjure violence;

(j) to strive towards excellence in all spheres of individual and collective activity so that the

nation constantly rises to higher levels of endeavour and achievement.

(k) to provide opportunities for education by the parent the guardian, to his child, or a ward

between the age of 6-14 years as the case may be

SOURCE:

Indian Polity – Laxmikanth

A SERIES

135

99. Consider the following statements:

1. Sikkim does not have mica mine.

2. Ladakh and Jammu and Kashmir does not have copper mine.

3. Silver mines are only found in Jharkhand and Odisha.

Which of the statements given above is/are correct?

(a) 1 only

(b) 2 only

(c) 1 and 2 only

(d) 1, 2 and 3

100. Recently, the Ministry of External Affairs had announced for the setting up of

New Emerging and Strategic Technologies (NEST) division to collaborate with the

foreign partners. Which of the following technologies will be covered under

“NEST”?

1. Artificial Intelligence

2. Internet of Things

3. 5G mobile generation

4. Quantum computing

5. Block Chain Technology

Select the correct answer using the code given below.

(a) 1, 2 and 3 only

(b) 1, 3 and 4 only

(c) 2, 4 and 5 only

(d) 1, 2, 3, 4 and 5

EXPLANATION:

The External affairs ministry announced the setting up of new emerging and strategic

technologies (NEST) division. The division will work to increase investment coordination

among the states of the country and also with the foreign countries.

NEST will act as the nodal division within the ministry for issues pertaining to new and

emerging technologies. It will help in collaboration with foreign partners in the field of 5G

and artificial intelligence. Quantum computing and Block Chain Technology are different

A SERIES

136

technologies that are not covered under NEST.So, option (a) is correct.

ADDITIONAL INFORMATION:

NEST and it’s functions:

The division will act as the nodal point in India’s foreign ministry for all matters

connected to new and emerging technologies including exchange of views with

foreign governments and coordination with domestic ministries and departments.

It will also help assess foreign policy and international legal implications of emerging

technology and technology-based resources.

The desk will also be involved in negotiations to safeguard Indian interests at

multilateral fora like the United Nations or the G20 where rules governing the use

and access to such technologies could be decided.

SOURCE:

https://economictimes.indiatimes.com/news/politics-and-nation/mea-sets-up-

emerging-technologies-division/articleshow/73063773.cms